Hocam mezun olduğumuz üniversite akredite değilse denklik belgesi diye bir belge varmış. Fakülteden talep edebiliyormuşsun. Denklik belgesi akredite belgesi ile aynı işlevi mi görüyor tam olarak nedir nasıl alınır
Merhaba Denklik belgesi yurt dışında fakülte bitirenlerin mesleği Türkiye'de yapmak istediklerinde diplomalarını geçerli saydırmak için YÖK e başvurarak almaya çalıştıkları belgedir bu belgeyi almak için tıp fakültesi mezunları ÖSYM müracaat eder ve STS seviye tespit sınavına girerler, sınavda başarılı olanlara YÖK tarafından denklik belgesi düzenlenir. USMLE sınavı ise Türkiye'deki tıp mezunların Amerika'da denk sayılabilmesi için yapılan sınavdır, akreditasyon ise bu sınavlara girebilmeniz için ABD nin tıp fakültelerinin yeterliliği ile ilgili uyum kriterine verdiği isimdir. Üniversitelerin akreditasyon taleplerini ABD tarafından yetkilendirilmiş kişi ve kuruluşlar denetler ve onaylar. Denklik belgesi ile akreditasyon belgesi birbirinden ayrı kavramlardır ve aynı işlevi yaptığından söz etmek mümkün değil. Başarılar dilerim. Ahmet Şahin
merhaba hocam,ilk once bilgisayarimin klavyesi turkce olmadigi icin kusura bakmayin lutfen. bi iki tane sorum var cevaplarsaniz sevinirim. 1.Bildigim kadariyla usmle toplam 4 sinavdan olusuyor, 2si ankarada ,3 u amerikada, birinci sinavi gecince 7 seneye kadar gecerli dediniz, peki 3. veya 4.sinavi gecemezse seneye hangi sinavdan girebilecek yani tekrar kaldigi sinavdan mi girmeye basliyacak YOKSA tekrar 2.sinavdan mi baslayacak Yani 2. 3. 4. sinavlarin da TIPKI 1.sinav gibi belli bir gecerlilik suresi var midir? 2.sorum:USMLE'nin tum 4 sinavini da gectikten sonra istegimiz bolume uzmanlik icin hangi asamalari gecmemiz gerekiyor? TESEKKURLER
Merhaba,Bilgilendirmen için teşekkürler. Sınavlarım ikisi Ankara’da diğer ikisi Amerika’da yapılıyor her birinin geçerlilik süresi yedi yıldır yedi yıl bitiminde daha önce girdiğiniz sınav bir sonraki Sınav için gerekli basamağı oluşturamaz yani geçerliliğini yitirir. Çalışmalarınızda başarılar dilerim Ahmet Şahin.
Merhaba, doğrudur, mevcut durumda step 3 devam ediyor, step 2 CS ise belirsiz bir süre için kaldırıldı, bunun yanı sıra, gerek puan hesaplama gerek sınav formatı gerekse sınava hazırlanmak amaçlı dijital kütüphane (online) gibi çalışmalar olduğu ifade ediliyor ancak bu değişikliklerin hangi zaman diliminde hayata geçirileceği ile ilgili henüz netleşmiş bir tarih söylenmiş değil. Başarılar dilerim.
Türkiyede uzman olduktan sonra usmle ye girebiliyor muyuz hocam ? Bir de amerikada doktorluğa başladığımız zaman orda belirli bir süre kalma zorunluluğumuz var mı, mesela 10 yıl sonra tekrar Türkiyeye dönebilir miyiz
Akredite olmuş bir fakülteden mezun iseniz Türkiye de uzman olduktan sonra da USMLE ye katılmanda bir engel yok. ABD de sınavları kazanıp uzman olarak çalışmak isterseniz bilmeniz gereken kontratları yıllık yapıldığı yani isterseniz kontrat bitiminde dilediğiniz yere gitme şansınız var. Başarılar dilerim.
Hocam öncelikle biz bu sınavları geçip asistan olarak göreve başladıktan sonra uzmanlığımızı amerikada mı alacağız ve asistanlıktan sonra amerikada çalışabilecek miyiz yoksa çalışma iznimiz iptal mi olacak
Evet doğru, asistanlık bitince uzmanlık belgenizi ABD den almış olacaksınız. Uzman olduktan sonra size sağlık profesyoneli olarak çalışabilmeniz için yıllık kontratlar teklif edecekler bu zaman diliminde vize türünüz J1 dan H1'a değişecektir. ABD’de sağlık profesyoneli olarak çalışabilmek için öncelikle ABD’de bulunan bir hastane ya da kliniğin işveren sıfatıyla sizi işe alması gerekmektedir. Bunun ardından, söz konusu işveren H1B vizesi almanız için sizin adınıza başvuru dilekçesi sunabilir. Çalışmalarınızda başarılar dilerim, Ahmet Şahin.
Şuanda ben 11.sınıfım seneye sınava giricem sizce hangi üniversite bize daha çok fayda sağlar akredite olması lazım dediniz bir de. Mesela Marmara üniversitesinin akreditasyonu bitecekmiş birkaç yıla bittikten sonra ne olur ? Çapa mı Cerrahpaşa mı yoksa Marmara mı bizim önümüzü daha çok açar? Sınavlara girdik diyelim yani USMLE ye kazansak hem step 1 i hem de step 2 yi sonra nasıl bir yol izlicez direk çalışabilecek miyiz? Bu süreçte neler yapmak bize fayda sağlar neler daha çok öne geçmemizi sağlar Amerika 'da doktorluk için?
Merhaba Yasemin, Üniversite anlamında sıralaman: Cerrahpaşa, Çapa ve Marmara şeklinde olabilir. Akreditasyonu biten üniversite büyük olasılıkla yeniliyor. Zor olan akredite olmak kolay olan süreyi uzatmak bu durum üniversitelere de prestij sağlıyor ve dolayısıyla hiçbir üniversite akreditasyonu iptal etmez. Cerrahpaşa dememdeki gerekçe ise: çalışma anlamında öğrencilerinin mümkün olduğu kadar önünü açıyor. Sınava nasıl çalışacağına gelince dönem birden itibaren temel bilimler ve temel bilimlerin TUS formatına dönem üçten sonra klinik bilimler ve günlük bilimlerin TUS formatını birlikte götürmen USMLE için kolaylık sağlayacaktır. Başarılar dilerim, Ahmet Şahin.
@@TUSEMANKARA_TUS Yanıtınız ve ilginiz için öncelikle teşekkür ederim. Bu dediğiniz 3 üniversiteden herhangi birini kazanıp USMLE ye girip onu da kazandıktan sonra bizi nasıl bir yol bekleyecek peki ? Direk çalışabilecek miyiz yoksa Amerika vatandaşı olmak gerekir mi uzun sure çalışabilmek için?
@@TUSEMANKARA_TUS Bir de aklıma takıldı biraz araştırma yapmıştım da Marmara nın tamamen ingilizce eğitim vermesi diğerlerine göre daha yeni olması gibi olanaklarının olması ile Cerrahpaşa dan ve Çapa dan daha iyi değil midir değişim programları da varmış baya emin olamadım ?
@@TUSEMANKARA_TUS merhaba, benim de bir sorum olacak. usmle'ye hazirlanmayi dusunen ve ingilizcesi cok iyi seviyede olan bir ogrenci icin turkce tip ve ingilizce tip tercihi arasinda bir fark var midir? tesekkur ederim.
Merhaba tıp türkçe 2. sınıf öğrencisiyim. İngilizce seviyem iyi ama tıbbi bilgim çok düşük.Sizce step 1'e nasıl hazırlanmalıyım? Tus dershaneleri yardımcı olur mu ? Yoksa kendim piyasadaki kitaplardan mı çalışmalıyım?Ve step1 sınavına ne zaman girmeliyim.3. Sınıfın sonu iyi midir? 6 senenin sonunda uzmanlık kazanırsam türkiyede dondurup usmle hazırlıklarına devam edebilir miyim? Hem usmle'e hem de tus'a mezun olana kadar hazırlanabilir miyim?
Merhaba, Step 1 basic sciences; tıp eğitiminin ilk üç senesindeki temel bilimleri kapsıyor demek mümkün bu durumda sınava 3 ün sonunda yada 5 in sonunda step 1-2 ye beraber girebilirsin unutmaman gereken özellikle step 1 den yüksek skor yapman kabul için avantaj sağlar. Kaynaklar için seçenek çok ister dersane kaynaklarından hazırlan ister USMLE World ve Kalan internet üyeliği ile sınavda benzeri olan sorulara ulaş veya internetten küçük bir araştırma ile kendine en uygun sitelerin kısa veya orta vadeli üyeliklerini tercih edebilirsin. Son soruna ise olumlu yanıt veremem Türkiye de uzmanlık dondurmak mecburi sebeplere dayandırılmalı raporlu sağlık sorunlar, doğum vb kısaca ikisine bir arada çalışılabilinir ancak ikisi bir arada yürütülemez. Ayrıca Türkiye deki uzmanlık ABD geçerli sayılmıyor. Tersi durum Türkiye için geçerli. Başarılar dilerim. Ahmet Şahin
Saygıdeğer hocam öncelikle yapmış olduğunuz videonuz için çok teşekkür ederim. Benim bir sorum var: Tıp fakültesine 21 yaşında başlamış olmam bir yıl hazırlık + altı yıl tıp sonucunda 28 yaşında mezun olmuş olacağım. 28 yaşımda mezun olacak olmam ABD'de doktor olmak için bana bir dezavantaj oluşturur mu?
Merhaba, ben mezuniyet yaşınızın size dezavantaj olacağını düşünmüyorum 37 yaşında olup Amerika'ya giden olduğunu söylemem sanırım yeterli olur. Asıl olan sizin kendinizi nerede gördüğünüz, güveniniz kırılmasın. Başarılar dilerim. Ahmet Şahin
Hocam, step1’i mezun olduktan hemen sonra vermek istiyorum. Ama step 2 yi 2 sene sonra vermek istiyorum(Amerikaya yerleşmek, dili daha iyi öğrenmek ve para biriktirmek için). Bu mümkün mü? Bana eksileri olur mu?
Merhaba, evet mümkün ve dil yeterliliği konusu doğru düşünülmüş nihayetinde diğer steplerde çok ihtiyacın olacak, step 1 verdikten sonra yedi yol geçerlilik süren var dilediğin gibi kullanabilirsin. Başarılar dilerim. Ahmet Şahin
Hocam video harika olmuş, netleştirmek için soruyorum da, step 1 ve 2 bittikten 3 ü vermeden sonra direkt olarak Amerika'ya yerleşip asistan olarak başlayabiliyoruz değil mi? Ve hangi üniversitede asistanlığa başlayacağımız nasıl belirleniyor? Sınavları verdikten sonra tek tek okullara mı başvuruyoruz, kabul almama ihtimalimiz de var mı? ve herhangi bir asistanlık ücreti vize oturma izni vesaire çok uğraştırır mı?
Merhaba, öncelikle sürecin oldukça maliyetli ve zorlu olduğunu ve sonuca ulaşmanın çok kolay olmadığını tekrar belirtmek isterim, sorunuza gelince USMLE Step 3 Sınavı, USMLE serilerinin son sınavıdır. Bu sınavın diğerlerinden oldukça farklı özellikleri vardır. USMLE Step 3 sınavı, ABD’de tıpta uzmanlık eğitimine giriş için bir önkoşul değildir. USMLE Step 1, Step 2 CS ve Step 2 CK sınavları alan ABD’de tıpta uzmanlık yoluna girebilmeleri için yeterliyken, USMLE sınavlarının son basamağı olan Step 3 sınavı ABD’de kısıtlanmamış (unrestricted full medical license) eyalet lisansı alabilmek için gerekli bir sınavdır ve asistanlığa başlandıktan sonra alınabilir. Her eyalette tıbbi lisans alma koşulları farklı olduğundan, Step 3 ile ilgili değişik düzenlemeler vardır. Başarılar.
hocam, amerikada uzman olduktan sonra türkiyeye nasıl dönülür, tusa girmek gerekir mi? ya da önce türkiye de uzman olup amerikaya gitmek istesek usmleye girmemiz gerekir mi, girersek de türkiyede uzmanlığımızın olduğu için kabul daha kolay olur mu?
Merhaba ABD de yada Yurtdışında tıpta uzmanlık eğitimi almış doktorların Türkiye’ye döndüklerinde de denklik almaları gerekmektedir. Bu durumda ise yurtdışında denklik için gidilen kurumun tanınmışlık listesinde olup olmaması, Uzmanlık eğitiminde geçirilen süre gibi farklı etkenler söz konusudur. Bu durumlara göre farklı denklik prosedürleri işlemektedir. Uzmanlığını bitirdikten sonra dönmeyi düşünen ya da dönme yolunu açık bırakmak isteyen hekimlerin, uzmanlaşmak istedikleri alanın Türkiye’deki durumunu da dikkatle incelemeleri çok önemli bir basamağı oluşturur. Ülkemizdeki uzmanlık mevzuatı ve denklik kriterleri cumhuriyet tarihimiz boyunca birçok kez değişikliğe uğramıştır. Ufak farklılıklar genel olarak aşılabilse de, uzmanlık eğitimi süresindeki büyük farklılıkların aşılması mümkün olmayabilir. O nedenle ABD’de uzmanlık yapmak isteyen hekimlerin bu yola girmeden önce , seçtikleri uzmanlık alanının bu yönden karşılaştırmasını mutlaka detaylı bir biçimde yapmaları gerekir. Sorunun ikinci kısmını cevabı ise evet Türkiye'de uzmanlık alanlar ABD de yeniden uzmanlık almak durumunda, kısaca kabul etmiyorlar diyebiliriz.. Başarılar dilerim. Yeni videomuzda Amerika maaşları inceleyecegiz, takipte kalabilrsin, Ahmet Şahin
Hocam anlamadığım bir konu var, usmle neticesinde yurtdisinda uzmanlık yaparken halihazirda doktor olarak çalışmıyor muyuz? Onların bize maaş vermesi gerekmiyor mu? Arkadaşların sorduğu eğitim ücreti nedir tam olarak
Merhaba, USMLE yi geçip asistanlığa başlarsanız aynen ifade ettiğiniz gibi onlar size ücret ödüyor, arkadaşlar sanırım Amerika'da tıp okumanın veya USMLE sınavlarına girmenin maliyetini sormuş olabilirler. Başarılar dilerim; Ahmet Şahin
Hocam diyelim ki bu sınavları geçtik hepsini ama Amerika'da referans olarak gösterebilecek bir tanıdığımız yok. Bu durumda referans olarak bulabileceğimiz kişileri nereden bulabiliriz?
Bu kişilere internetten ulaşabilir veya yurt dışı öğrenci programları yapan şirketlerden yardım alabilirsiniz. Bu sınavları verene kadar mutlaka bir referans bulacağınıza inanıyorum. Ahmet Şahin.
Ömer bey, Amerikan kültür web sitesi yada arayarak bir bilgi kaynağı yada tarih sorabilirsiniz, usmle resmi sitesinde gerekli zamanlarda bilgi veriliyor.
Merhaba, güzel bir soru ancak bu soruya yanıt verebileceğim bir istatistik yok elimde maalesef. Ancak tahminde bulun dersen dörtte bir diyebilirim. Başarılar dilerim. Ahmet Şahin
Çok teşekkür ederiz doğru düzgün bir USMLE tanıtım gördük sayenizde aklıma takılan 1-2 soru vardı yanıtlarsanız sevinirim Amerika’da girilen 2 sınav arası süre hakkında bilginiz var mıdır bir de referans mektubunu getirip sınavı geçtikten sonra ne zaman çalışmaya başlayabiliyoruz yanıtlarsanız çok memnun olurum saygılar
Merhaba ; Sınavlarınızı, belirli bir sistematiği olmakla beraber, kendi belirlediğiniz bir tarihte alabilirsiniz. Herhangi bir sınav sonucunuzun geçerliligieğeç geçtiyseniz yedi yıldır ve bu süre içinde aynı sınavı bir daha alamazsınız, geçemediyseniz yineleyebilirsiniz. ihtisas başvurusu her sene bir kez yapılır (Genelde Eylül 1 ile başlayan iki- üç aylık bir dönemde). Başlama ile ilgili tarih sizin başvurularınız ve bu başvurular için görüşme ve kabul aşamasından sonra gerçekleşir. Sorduğunuz bu zaman kişinin girişim becerisi ile uzayıp kısalabilir. Ancak görüşme öncesi dönem, görüşme günü ve görüşme sonrası bu dönemin asıl belirleyicisidir.Başarılar.
@@furkanemrecuhadar4546 Türkiyede aldığınız uzmanlık USA da uzmanlıgı geçerli kılmaz ama kabül edilmenizde artı bir madde olarak işlev görür. Uzmanlıgı olmayan birine gore tercih sebebi olursunuz. Ayrıca Türkiyede Uzmanlık yapmak B planı olarak size bir güven verecek ve USMLE de beklediginiz sonucları alamazasınız burada bir garantiniz olacaktır, Başarılar.
merhaba tum adımları geçip gereken her şeyi tamamladığımız takdirde boşta kalma ihtimalimiz var mı acaba?en azından bir yerde işe başlamamız kesin mi yoksa hocam
Merhaba, tüm adımları geçmeniz gerek şart asıl olan müracaat yaptığınız yerlerden alacağınız olumlu yanıt işte bunun garantisi yok. Kısaca tüm sınavları vermek sizi mutlak iş sahibi yapar demek mümkün değil. Başarılar dilerim. Ahmet Şahin
Iyi gunler .sizi yeni izledim.burada uzman olan biri icin orda uzman olarak calismanin bir yolu varmidir? Ve ya universite mezunu ile uzman doktor arasinda bir fark varmidir asistanlik hayatinda ve ya sinavinda?
Merhaba, burada uzman olan biri için orda uzman olarak çalışmanın yolu videoda anlatılan sınav sisteminden geçmeniz ile mümkün. üniversite mezunu ile uzman doktor arasında fark sorusunun yanıtı, sınav sistem ve asistanlığa ulaşmak için izlenen yol anlamında hiçbir fark yok ancak asistanlık kabulünde öncelik sebebi olabilir, Türkiye'den uzmanlık alanların fazladan sahadaki tecrübe ve edinimleri asistanlıkta avantaj olacaktır. Başarılar. Ahmet Şahin
Merhaba hocam. Tusta fizyoloji, sinirbilim, mikrobiyoloji gibi temel bilim bölümleri seçilebiliyor. Anladığım kadarıyla USMLde bu yok. Türkiyede tıp okuduktan sonra ABD'de temel bilimlerden devam etmenin yolu nedir?
Merhaba, Öncelikle şu konuyu açıklayalım. Temel bilimlerde asistanlık diye bir kavram dünya da Türkiye'den başka bir ülkede yoktur. Türkiye temel bilimlere de TUS ile "asistan" almakta ve "doktora" programları haricinde bir yan yol yaratarak temel tıp bilimleri "uzmanlığı" gibi tuhaf bir kategori yaratmaktadır. Bu problem nedeniyle, dünyada fizyoloji uzmanlığı, biyokimya uzmanlığı gibi kavramlar anlaşılamamaktadır çünkü bu yollar tıpta akademik geleneklere aykırıdır ve anlamı yoktur. Fizyoloji, biyokimya, anatomi, histoloji, farmakoloji gibi bölümlerin "asistanlığı (residency)" ve uzmanlığı olmaz. Bu bölümlere girebilmek için "doktora (PhD) Doctor Of Philosophy" programları olur ve bitirene o konuda doktora verilir. Başarılar dilerim. Ahmet Şahin
Hocam ve değerli arkadaşlar merhaba. Türkiyede uzman olduktan sonra, Amerikada aynı uzmanlığı yapmak için yine aynı sınavlardan geçiyoruz ve tekrar orada sıfırdan asistanlığa başlıyoruz değil mi? (yani burada uzmanlığı bitirmemiz bizim oradaki sürecimizi kısaltmıyor??) Yanlış biliyorsam bilenler aydınlatsın lütfen. Teşekkürler.
Merhaba Türkiye’de uzmanlık alan arkadaşlar Amerika’da uzmanlıklarını saydıramıyorlar. Asistanlık sürelerinden tedavi yöntemlerine kadar birçok fark neden gösteriliyor dolayısıyla her branşta baştan ihtisas istiyorlar. Başarılar dilerim, Ahmet Şahin
Malum ucretler biraz fazla. Sinava girmek için belli bir süre var mı? yani ben mezun olduktan sonra da girebilir miyim yoksa illa okurken mi girmem lazım?
Merhaba, çok haklısın ücretler konusunda, sınava belli bir sürede katılma zorunluluğunuz yok tabi ki ancak step 1 ve step 2 için mevcut okul bilgilerinin avantaj sağladığını bilmeniz yararlı olabilir, bunun dışında sınava mezun olduktan sonrada girebilirsiniz. Başarılar dilerim. Ahmet Şahin
@@TUSEMANKARA_TUS Hocam peki diğer ulkeler için de böyle icerikleriniz var mı? mesela Almanya'yi arastirdim. Bu şekilde bir sınavdan ziyade dil ön plana çıkıyor.(İngilizcem b1-b2 seviyesinde ama almancam hiç yok) diğer ülkelerle ilgili icerikler de gelebilir mi?nasıl gidilir, ne kadar para gerekli, bu parayı burs olarak alabilecegimiz bir yer var mı, tip için dili nereden ogrenebiliriz vs. tarzı. mesela kanadayi arastirinca gördüm ki doktorlar rahat ama oraya gidip doktor olarak calismak çok zormus.
Çok iyisinizz çok iyi videolarrr çok iyisinizz vücutlarınızz çokk iyii çok samimisinizzz çok iyi yayıncılarsınız ve çok iyi TH-camrlarsınız çok iyi oyuncularsınız MAŞALLAH çok yakışıklısınız ve çok güzelsiniz sizler buradaki insanlar gibi çok güzelsiniz ve çok yakışıklısınız vücutlarınızz çokk iyii teşekkürlerr ❤️❤️ çokk iyii vücüdunuzz varr çokk başarılısınızz çokk zekisinizz MAŞALLAH sizler buradaki insanlar gibi çokk başarılısınızz çokk zekisinizzz MAŞALLAH çok teşekkürler her durum için❤️❤️❤️❤️ İNŞALLAH ALLAH tüm insanların gönlüne göre versin AMİN
Değerli hocam eğer kısıtlanmamış eyalet lisansı almazsak yani step 3'e girmesek ABD'de doktorluk yapamaz mıyız ? Cevaplarsanız çok müteşekkir olurum şimdiden teşekkürler.
Hocam nöroloji asistanlığı için Amerika'dan referans mektubu almak şart mıdır ? Yüksek USMLE puanı ile mektupsuz match alamaz mıyız ? Bir de referans mektubu şart ise kaçıncı sınıfta hangi şekilde (clerkship vb.) almalıyız ? Cevaplarsanız çok sevinirim hocam.
Merhaba, referans mektubu tüm asistanlıklar için gerekli sadece yüksek puan yeterli olmaz tüm bunlarla beraber müracaat yapacağınız yere detaylı bir öz geçmiş hazırlamanız sizden beklenenler arasında yayınlanmış en az birkaç makaleniz de varsa CV de belirtin tüm bunlar asistanlığa kabulünüz için garanti sağlamaz ancak kabul için avantaj sağlayacaktır. Başarılar dilerim. Ahmet Şahin
Merhaba, bu konuda öngörüde bulunmak kolay değil ancak görüne o ki özellikle Türkiye gibi ülkelerden gidecek doktorların CV deki en önemli kozlardan bir tanesi gidecek gibi görünüyor, tabi bu kararın temelinde ABD li öğrencilerin bu yükten kurtulma planı olduğunu söylemek gerek. Türkiye'den gidecek öğrenciler için demek oluyor ki CV nin yayın ve ABD stajları bölümü daha değerli olacak. Başarılar dilerim.
Hocam uzman doktor maaşlarını 400bin dolar diye okudum acaba vergi kesintisiz hali midir yoksa bu meblağdan Yüzde 60falan vergi mi kesilicek?Ve sınav dışında yurt dışına gitme olasiligim düşük gibi duruyor referansım da yok acaba sıkıntı olabilir mi veya nasıl halledeilebilir ?cevaplayabilirseniz çok sevinirim
Merhaba, Genel olarak yurt dışındaki ücretler yıllık ve bürüt üzerinden konuşulur, yani elinize geçen net rakam değildir. Referans olmaması sorun olur tanıdık veya staj şansı ile bir şekilde referansı halletmen gerekir. Ahmet Şahin
%30 oranında vergi var. %60 değil. Ünsanlar Amerika'daki vergi oranını nedense çok abartıyor. Zanneden de maaşının %90'ı vergiye gidiyor sanıyor. Ama öyle birşey yok. Üstelik sene sonlarında vergi geri ödemesi alıyorsunuz. Bekarsan%30, evliysen%40 çocuğun varsa %50 oranında vergi geri ödemesi var.
Merhaba, Türkiye’de tıp okuyup USMLE sınavlarını geçerseniz ABD de eyaletlerde doktorluk yapabilme hakkı elde ediyorsunuz bu belge ile dilediğiniz eyalete müracaat edebilir ve kabul görürseniz aile hekim (pratisyen) olarak mesleğe başlayabilirsiniz. Başarılar dilerim.Ahmet Şahin
@@TUSEMANKARA_TUS Aile hekimliğini seviyorum hemde diğer uzmanlıklardan daha rahat çalışma konusunda. Yurt dışında aile hekimliği uzmanlığı yapıp ardından uzman olarak çalışabiliyormuyuz yani? İntarnette hiç bilgi görmedim aile hekimi olarak çalışan biri(yurt dışında)
Merhaba, on dokuz mayıs üniversitesinin akreditasyonu var olarak biliyorum ancak emin olmak için okul dekanlığından ve imedidata.com sitesinden kontrol edebilirsin. Başarılar dilerim. Ahmet Şahin
@@TUSEMANKARA_TUS turkce programinin varmis ama ingilizce programi yeni basvurmus akreditasyona (2019) . 6 yil icerisinde alabilir mi acaba akreditasyon?
Merhaba, USMLE sınavı veya daha doğru bir deyimle “sınavları”, birinci basamak (Step 1), ikinci basamak teorik (Step 2 Clinical Knowledge), ikinci basamak klinik beceri (Step 2 Clinical Skills), üçüncü basamak (Step 3) olmak üzere 4 adet sınavın ortak adıdır. Bu sınavların her biri Amerika Birleşik Devletleri’nde herhangi bir eyaletin tıp doktoru lisansını alabilmek için (bağımsız pratik yapabilme hakkını elde edebilmek için) gerekli sınavlardır. Bu sınavların herhangi birini giren bir kişi hiçbir kadro alamaz, sadece gerekli basamaklardan birini tamamlamış sayılır. Diğer deyişle, USMLE sınavlarının tamamını tamamlamayan ne bir asistanlık programına başvurabilir, ne de eyalet lisansı alabilir. ABD’de asistanlık/uzmanlık eğitimine başvuran bir hekim, USMLE Step 1, Step 2 CK, Step 2 CS sınavlarını başarıyla geçmiş olmak zorundadır. ABD’de bir eyalette bağımsız çalışma izni almak isteyen bir hekim ise bu 3 sınava ek olarak USMLE Step 3 sınavını da başarıyla geçmiş olmak zorundadır. Dikkat edilirse, USMLE sınavlarının herhangi birisini ya da tamamını başarıyla geçmiş olmak, ABD’de hiçbir kadro garantisi vermez, hiçbir hastanede çalışmaya başlamak için bir pozisyonu sınav sonucuna bağlı olarak sunmaz. Ahmet Şahin.
Yurt dışı seçeneklerini değerlendirecek öğrenciler için altın değerinde bir video olmuş. Emeklerinize sağlık, teşekkürler. Ayrıca referans mektubu hakkında bir videonuz olursa çok mutlu olurum :)
Bora bey Merhaba, faydası olduysa ne mutlu. Referans mektubu USA'da staja gidildiginde o klinikte ki bir hekimden alınabiliyor. Aynı zamanda dünya çapında ünlü USA'da görev yapan bir cok Türk hekimi ünlüleri dahil olmak üzere mümkün olduğu oranda rereferans olma noktasında yardımcı olabiliyor. Biraz sosyal iletişim biraz doğru adresle çok zorlanılan bir konu olmuyor. Bu konu tek bir videoyu doldurmamız zor ancak sıkca sorulan sorular videosunda yer verilecek sorulardan biri olabilir, katkınız ve ilginiz için teşsekkürler. Nicelerinize referasn olmanız dilegiyle, hoşcakalın.
@@ornitorenkmafya2900 brüt olduğunu herkes biliyor. Vergisini çıkarınca yine de yüksek çıkıyor. Yorumunuzu okuyan da net maaşın %10 verginin%90 olduğunu zannedecek. Yorum yaparken düzgün yapın.
@@TUSEMANKARA_TUS kazançlar değil mesele onlar sadece kendi milletlerine güveniyorlarmis ameliyat konusunda yani Amerikalı olmayan biri kalp beyin genel vb cerrah olamıyormus Amerika'da . Yani usmle sınavı bittikten sonra kişiler hangi uzmanlıkların asistanlığına girebiliyor
Hocam merhabalar. Hacettepe tıp 1.sinifim. Mezun olduktan sonra Amerika'da çalışmak istiyorum. Onun için her gün çalışıyorum ama bu sınavı kazanabilmek için günde kaç saat çalışmak gerekir
Merhaba, USMLE kazanmak için evet günde 1-3 saat ile başlayıp 5 sınıfın sonuna doğru 6-8 saate çıkabilirsen ciddi mesafe kat edersin ancak ne kadar çalıştığın sorusuna nasıl çalıştığın sorusunu da eklemelisin. Öncelik temel bilimler ve İngilizce iyileştirmesi olmalı 4. Sınıf ile klinik bilim eklemelisin. Başarılar dilerim. Ahmet Şahin
Hocam merhaba sabırla herkese cevap vermenizden cesaret alarak ben de sizden bilgi almak istiyorum ve çok teşekkür ederim. 6 aydır tıp fakültesinden mezunum pratisyen hekim olarak çalışıyorum 25 yaşındayım. ABD de doktorluk yapma şansımın olduğuna ikna oldum diyebilirim. Ve şartlarımı zorlamak istiyorum . Dil sevyem B2 . Sizce dili geliştirirken bir yandan da USMLE kaynaklarına mı hazırlanmalıyım ? Yoksa direkt USMLE kitaplarını tekrar ederek step 1 ve CK i geçebilir miyim ? En efektif çalışma nasıl olur bilmiyorum. Çevremde destek alacağım birileri yok. Bir yandan pratisyenlik yapıp bir yandan da çalışabilirim değil mi? TUS sınavı da iyi bir sonuç için çok efor istiyor. Uzun soluklu düşününce USMLE benim isteklerimi karşılıyor. Sadece yol yordam öğrenmek istiyorum. Dönerseniz çok memnun olurum. Saygılarımla
Kıymetli hocam bir de istediğiniz konu varsa paylaşın demişsiniz. USMLE step 1 ve 2 yi geçtikten sonra başvuruların kabulü için referans mektuplarına ihtiyacımız var bunun için de orda çalışmamız bu kısır döngü gibi geldi. Referans mektupları için yine kendimiz masraflarını karşılamak üzere orda bir üniversite gözlemci olarak çalışabilirmişiz . Ama bunun için nasıl kabul alacağım? CV de ne gibi şeyler sunmam gerek ki tabi gel gör desinler ?
Tekrar merhaba, bununla ilgili iki seçenekten ilki orada yaşayan ve çalışan Türk doktorlardan birilerine ulaşıp yardım alman, ikincisi ise öğrenci değişim programları yapan firmalardan ABD ye çalışan bir kurum bulman uygun olabilir. İlave olarak bir üçüncü yol hangi branşta gözlemci olmak istiyorsan mail yolu ile müracatta bulunup kabul bekleyebilirsin.kolaylıklar başarılar.... Ahmet Şahin
Merhaba birkaç aşamada yanıt vermeye çalışacağım öncelikle hedef USMLE ise gerçekten çok iyi derecede İngilizce alt yapısına sahip olmak gerekir. Şöyle düşünme step 1 ve step 2 CK yı vereyim CS ve sonrası için dile yüklenirim bu yanlış olur çünkü step1 için yüksek puan gerekli olacak dolayısıyla iyi İngilizce orda da gerek. Etkili çalışma için tek bir yöntemden bahsetmek doğru olmaz bu biraz kişisel ancak bana sorarsan, en uygun yöntem TUS çalışır gibi çalışman ve konu tekrarlarının ardından USMLE soruları çözmen bunu da her step için ayrı ayrı yapman ve tabi bol tekrar uygun olur. Başarılar Ahmet Şahin
Hocam merhaba, sınavlar bittikten sonra başvurmak dışında asistan( yani maaş almaya başlamak) olmamız için yapmamız gereken başka bir prosedür var mı ?
Merhaba, USMLE Step 3 Sınavı, USMLE serilerinin son sınavıdır. Bu sınavın diğerlerinden oldukça farklı özellikleri vardır. USMLE Step 3 sınavı, ABD’de tıpta uzmanlık eğitimine giriş için bir önkoşul değildir. USMLE Step 1, Step 2 CS ve Step 2 CK sınavları alanABD’de tıpta uzmanlık yoluna girebilmeleri için yeterliyken, USMLE sınavlarının son basamağı olan Step 3 sınavı ABD’de kısıtlanmamış (unrestricted full medical license) eyalet lisansı alabilmek için gerekli bir sınavdır ve asistanlığa başlandıktan sonra alınabilir. Her eyalette tıbbi lisans alma koşulları farklı olduğundan, Step 3 ile ilgili değişik düzenlemeler vardır. Başarılar.
Merhaba, tıp fakültesine gelen arkadaşların büyük bölümü fen lisesi ve anadolu lisesi mezunları yabancı dil düzeyi çoğunda sizden farklı değil ve hepsi üniversitede ve devamında bu problemi çözüyorlar. Merak etmeyin kısa zamanda aynı kelimeleri duya duya alışıyorsunuz hatta bazen Türkçe karşılığını bulmakta zorlandığınız için günlük hayatta bu kelimelerden istemsizce yardım alıyorsunuz. Başarılar dilerim. Ahmet Şahin
ilk önce size çok teşekkür ederim, çok faydalı bir video oldu :) benim bir sorum var ; mezun olduktan sonra sadece step 1 ve step 2’yi yaptığımızda ve hem puanı yüksekse de referans mektubumuz varsa step 3 olmadan kabul edilebilir miyiz?ve uzmanlığa girip sonra step 3’ü bitirmemizlazım mı ? Birazcık kafa karıştırıyor 😄 . sınava 4.sınıfta hazırlamak ve 5.dönemin sonunda step 1 ‘e girmek geç mi hocam? Tekrardan teşekkürler 🙏🏻
Merhaba Dotchi Odeh ; USMLE Step 3 Sınavı, USMLE serilerinin son sınavıdır. Bu sınavın diğerlerinden oldukça farklı özellikleri vardır. USMLE Step 3 sınavı, ABD’de tıpta uzmanlık eğitimine giriş için bir önkoşul değildir. USMLE Step 1, Step 2 CS ve Step 2 CK sınavları alanABD’de tıpta uzmanlık yoluna girebilmeleri için yeterliyken, USMLE sınavlarının son basamağı olan Step 3 sınavı ABD’de kısıtlanmamış (unrestricted full medical license) eyalet lisansı alabilmek için gerekli bir sınavdır ve asistanlığa başlandıktan sonra alınabilir. Her eyalette tıbbi lisans alma koşulları farklı olduğundan, Step 3 ile ilgili değişik düzenlemeler vardır. Dönem 5 sonu step 1 girmek konusu ; ne kadar erken o kadar iyi, ne kadar geç o kadar zor olur. Yinde kişisel tercihe kalmış bir durum. iyi Çalışmalar. Başarılar...
Hocam aile hekimi olmak için step 3'e girmemiz gerekiyor mu ve yine aile hekimliği için referans mektubu gerekli mi ? Cevaplarsanız çok sevinirim şimdiden teşekkürler.
hocam 11.sınıfı bitirip türkiyeden ingiltereye taŞındım ve ib sistem olarak 2 yıl okuyacağım sonrasında amerikada üniversite okumak istiyorum estetik cerrahiyi planlıyorum sat ye mi girmem gerekecek kaç yıl okuyacağım anlatabilir misiniz
Merhaba, Community College olarak adlandırılan iki yıllık okullara giriş şartları dört yıllık üniversitelere göre daha esnektir. Bu okullar SAT veya ACT sınav sonuçları istemezler. Ancak, değişmedi ise Dört yıl ve üzeri eğitim süreleri için üniversiteler için gerekli olarak biliyorum.
Merhaba, Faydalı olmasına sevindik. TUS ve Eksenindeki konular için Kurumumuza her zaman uğrayabilirsiniz , tanışmak icin randevulu gelirseniz daha garanti olur, iyi çalışmalar...
Hocam iyi günler Ben de yurtdışı düşünüyorum şuan üniversite tercih dönemindeyim.Şimdi benim şöyle bir sorunum var puanımın yettiği ve akredite olan ünilerin ingilizce hazırlığı yok ve benim İngilizcem 0 sizce tıpı Türkçe okusam kendimi bu sınavlara yeticek kadar ingilizce öğrenebilir miyim(İngilizcem berbat)
Merhaba, sıfır ingilizce ile başlayıp ileri düzey ingilizceye gelebilen bir çok örnek mevcut tabi ki sende yapabilirsin 1.ve 2. sınıfta yüklenmeye başlayıp USMLE sınavlarına kadar ileri düzeye gelebilmen mümkün. Başarılar dilerim.
Hocam ben 5. sınıfa geçiyorum, USMLE hazırlanmaya yeni karar verdim. TUS dershanesi başladığında dersleri dinlemem faydalı olacak mıdır yoksa sadece USMLE kaynaklarından mı çalışmamı önerirsiniz?
Merhaba. TUS dershanesinde dinlediğin dersler ve hatta biraz daha kolayı USMLE sınav soruları gibi düşünebilirsin, mutlaka faydasını göreceksin olur da fikrin değişir ve TUS a girmek istersen, eğitimin cebinde kalmış olur. Başarılar dilerim. Ahmet Şahin
Hocam merhaba.Türkiyede herhangi bir tıp fakültesinden mezun olunca bu sınavlara girmeye hak kazanabilir miyiz yoksa belli şartları var mı? Belirli üniversitelerden mi mezun olunmalı?
Merhaba, tek şart mezun olduğun üniversitenin akreditasyon sağlamış olması birçok fakültemiz bu durumu karşılıyor merak ettiğiniz fakülteleriimedidata.com adresinden sorgulayabilirsiniz. Başarılar dilerim. Ahmet Şahin
Tekrar merhaba, ABD de aynı dalda asistanlık yapabilmen için oluşturduğun dosyada CV kısmında Türkiye'de aynı bölümün asistanlığını yapmış olmanı belirtmen tercih sebebi olabilir, elini güçlendirir, daha kolay kabul görür. Hiçbir işe yaramaz demek mümkün değil ayrıca çokta zekice, ola ki bir gün Türkiye'ye dönmek istersen, hem ABD den hem de Türkiye'den uzmanlık almış olman, Türkiye'de tercih edilme konusunda lehine olacaktır,Ahmet.
Merhabalar hocam, Alküde okuyorum 1. Sınıfım şuanda akredite degiliz akredite olmak için ne yapılmalı üniversitenin akredite olmaması durumunda akredite bir üniversiteye yatay geçiş yapmam bir sorun teşkil eder mi usmle için
Merhaba, akredite olmayan ünv. mezunları için sınava giriş mümkün değil koşulları yerine getirmesi gerek üniversite yönetimi yok onlar yerine getirmiyor ben yatay geçiş ile akredite olan bir ünv. ye geçebilirim diyorsan olur nihayetinde diplomanın alındığı okulun akreditasyonu sorgulanıyor. Başarılar dilerim.
Merhaba, akredite olmayan ünv. mezunları için sınava giriş mümkün değil koşulları yerine getirmesi gerek üniversite yönetimi yok onlar yerine getirmiyor ben yatay geçiş ile akredite olan bir ünv. ye geçebilirim diyorsan olur nihayetinde diplomanın alındığı okulun akreditasyonu sorgulanıyor. Başarılar dilerim. Ahmet Şahin
Hocam ben şimdi 1.sınıf tıp öğrencisiyim diyelimki bu sınavları geçtim ABD'de asistanlığa başladım uzman olduktan sonra tekrar türkiyeye dönüp türkiyedr doktorluk yapmak istersem nasıl bir süreçten geçmem gerekiyor baştan uzmanlık yapmam gerekir mi? Eğer türkiyede yapılan step1 ve step2 sınavlarını geçıpte diğet sınavları geçemezsem sonra en başan tüm sınavlara girmek zorunda mıyım? Ayrıca bu sınavları kazanmak ne kadar zor? sonuçta çok pahalı sınavlar cevaplarsanız sevinirim şimdiden çok teşekkürler.
Merhaba ABD de yada Yurtdışında tıpta uzmanlık eğitimi almış doktorların Türkiye’ye döndüklerinde de denklik almaları gerekmektedir. Bu durumda ise yurtdışında denklik için gidilen kurumun tanınmışlık listesinde olup olmaması, Uzmanlık eğitiminde geçirilen süre gibi farklı etkenler söz konusudur. Bu durumlara göre farklı denklik prosedürleri işlemektedir. Uzmanlığını bitirdikten sonra dönmeyi düşünen ya da dönme yolunu açık bırakmak isteyen hekimlerin, uzmanlaşmak istedikleri alanın Türkiye’deki durumunu da dikkatle incelemeleri çok önemli bir basamağı oluşturur. Ülkemizdeki uzmanlık mevzuatı ve denklik kriterleri cumhuriyet tarihimiz boyunca birçok kez değişikliğe uğramıştır. Ufak farklılıklar genel olarak aşılabilse de, uzmanlık eğitimi süresindeki büyük farklılıkların aşılması mümkün olmayabilir. O nedenle ABD’de uzmanlık yapmak isteyen hekimlerin bu yola girmeden önce , seçtikleri uzmanlık alanının bu yönden karşılaştırmasını mutlaka detaylı bir biçimde yapmaları gerekir. Sorunun ikinci kısmını cevabı ise evet step 2 den sonra uzmanlık almak için diğer sınavlara girmen gerekir, barajı geçtiğin her sınavın geçerlilik süresi yedi yıldır. Diğer sorun ise evet sınavlar çalışmaya ve kişiye göre değişkenlik göstermekle birlikte kolay olduğu söylenemez ancak aynı zorluğun TUS içinde geçerli olduğunu söyleyebiliriz. Başarılar dilerim. Ahmet Şahin
hocam akreditesi olmayan bir üniversitede olursak veya girdikten sonra mezun olasıya kadar akreditesi biterse ve yenilemezlerse sınava giremiyoruz değil mi?
Video için teşekkürler.Ben Azerbaycanda 5 cı sınıf öğrencisiyim.Step 2 cs ankarada hazırlığı yapılıyormu?Yapılıyorsa ücreti ne kadar ?Diğer bir sorum referans mektubu için cerrahi branşda ne yapmamız gerekiyor?Yani mesela normalde hastaya dokunma iznimiz oluyormu o süreçde? Önceden teşekkürler
Merhaba Sorduklarınızın bir kısmı video içinde yüzeyelde olsa değinildi, Ankarada bu tip bir hazırlık yok, aşamalara internet üzerinden geçiş yapıp referans mektubu aşamasında mezun olmus olup , ikili ilişkilerle amerikadaki bir doktorun size referasn mektubu vermesi gerekiyor.Başarılarınız, diyaloglarınız doğru kişilerle iletisim kurarak mektubu talep etmeniz süreci hızlandırabilir. Başarılar dileriz.
Hocam merhaba akredite olöayan bir fakültesinden mezun kişi üniversitenin daha sonra akreditasyon alması spnucu usml girme hakkı kazanır mı ?(örneğin mezun oldukdan 5 yıl sonra üniversite akretitasyon almış )
Ücret i ödeyenlerin küçük bir yüzdesi bu sınavlarda başarı ve kabul yakalayabiliyor. Bir kesinliği yok sizin başarnıza bağlı. Ama kesiligi olsa evet hedefiniz yurtdışı ise değer bir ücret. Dİger yorum diş hekimligi demişsiniz, bu sınav sadece tıp hekimleri için diş hekimliği farklı onu araştırın istersiniz.
Hocam ben psikiyatrist olmak istiyorum Amerikada .bunun için anlattıklarınızdan başka bir şey yapmam lazım başka kaynak bulamadım cevaplarsanız sevinirim
Hocam diyelim tıp fakültesi 4. Sınıfta step 1'e girdik, mezun olana kadar step 2' ye girmeme gibi bir hakkımız var değil mi? 2 sene mezun olmak için bekleyebiliriz yani?
Merhaba Çok doğru step 1 sonrası, step 2 ye mezun olduktan sonra girebilirsin. Bilmen gereken sınavların geçerlilik süresi yedi yıldır, bu sürenin aşılmamasına dikkat etmelisin. Başarılar diliyorum, Ahmet Şahin.
Merhaba ABD de kaç yıllık uzman olduğunuza ve performansınıza göre değişiklik göstermekle birlikte 150 bin ile 750 bin dolar arasında yıllık kontratlar konuşulması mümkün.
merhaba, evet. Ancak akreditasyon zaman içinde alınabilir veya kaybedilebilir bir olgu. Yani şu an akredite olmayan br fakülte 3 sene sonra yeterlilikleri oluşturursa akredite olabilir
@@TUSEMANKARA_TUS yanıtınız için teşekkür ederim. Varsayalım ki akredite olmayan bir üniversiteden mezun olan bir öğrenci TUS'a girdi ve akredite bir üniversitede uzmanlık eğitimini tamamladı. Sonrasında herhangi bir nedenle Amerika'da çalışmak istediğinde bu mümkün olur mu? Olursa hekimi ne gibi engeller bekler?
Merhaba hocam, Türkiye'de ilk 6 senelik eğitimi akredite olmayan bir üniversiteden almış olup uzmanlığı ise akredite olan bir üniversitede okumuşsam yine de USMLE'ye girme hakkım olur mu?
Hocam sınavı kazanıp asistanlığı Amerika'da yapıp uzmanlığımızı orada alır ve çalışmaya başlarsak birkaç yıl sonra Türkiye'ye dönüp aynı uzmanlık burada geçerli olacak şekilde doktorluk yapabilir miyiz?
Merhaba ABD de yada Yurtdışında tıpta uzmanlık eğitimi almış doktorların Türkiye’ye döndüklerinde de denklik almaları gerekmektedir. Bu durumda ise yurtdışında denklik için gidilen kurumun tanınmışlık listesinde olup olmaması, Uzmanlık eğitiminde geçirilen süre gibi farklı etkenler söz konusudur. Bu durumlara göre farklı denklik prosedürleri işlemektedir. Uzmanlığını bitirdikten sonra dönmeyi düşünen ya da dönme yolunu açık bırakmak isteyen hekimlerin, uzmanlaşmak istedikleri alanın Türkiye’deki durumunu da dikkatle incelemeleri çok önemli bir basamağı oluşturur. Ülkemizdeki uzmanlık mevzuatı ve denklik kriterleri cumhuriyet tarihimiz boyunca birçok kez değişikliğe uğramıştır. Ufak farklılıklar genel olarak aşılabilse de, uzmanlık eğitimi süresindeki büyük farklılıkların aşılması mümkün olmayabilir. O nedenle ABD’de uzmanlık yapmak isteyen hekimlerin bu yola girmeden önce , seçtikleri uzmanlık alanının bu yönden karşılaştırmasını mutlaka detaylı bir biçimde yapmaları gerekir. Sorunun ikinci kısmını cevabı ise evet Türkiye'de uzmanlık alanlar ABD de yeniden uzmanlık almak durumunda, kısaca kabul etmiyorlar diyebiliriz.. Başarılar dilerim.Ahmet Şahin
Değerli hocam. Dönem 1 öğrencisi USMLE sınavına nasıl hazırlanabilir? Bir de eğer yurtdışında doktor olunursa mezuniyetten sonraki zorunlu hizmet görevi yapılma zorunluluğu devam ediyor mu? Şimdiden çok teşekkür ediyorum.
Merhaba, Dönem bir öğrencisi normal eğitimle devam eder iken aynı zamanda temel bilimlerden soru çözmeli dönem üçten sonra klinik bilimler eğitimi ile birlikte Klinik soruları çözmeli yani özetle tıpta uzmanlık sınavına çalışır gibi çalışabilirsiniz nihayetinde sınav tipleri soru anlamında birbirine çok yakın. Devlet hizmeti yükümlülüğüne gelince yurtdışında çalışacak isen yükümlülüğünü yerine getirmeden de gidebilirsin bağlayıcı bir durum yok. Sadece Türkiye’de olduğun süre içinde mesleği yapabilmen adına bu yükümlülüğü yerine getirmen bekleniyor. Başarılar dilerim. Ahmet Şahin
Merhaba, USMLE sınavı bilindiği gibi senin mezun olduğun tıp fakültesini diplomanın ABD de denk sayılması için yapılan sınavlar serisi bu seri bitirildikten sonra ister aile hekimi olmak üzere müracaat edebilir istersen diğer uzmanlık alanları için başvuru yapabilirsin belirtmeliyim ki bu sınavları geçmek veya başvuruları yapmak seni mutlak suret ile iş sahibi yapmaz. Asistanlığa kabul görmen durumunda yaklaşık dört yıl eğitim sürecinden geçersin fakat bu sürede aylık asistan ücreti almaya devam edersin. Başarılar dilerim. Ahmet şahin
Merhaba, step cs ve step3 e ABD de giriyor her ikisindede birebir hasta ile konuşma mülakatı yapılıyor bu bölümde daha çok İngilizce anlamanız ve konuşarak doğru yönlendirmeniz test esiliyor. Yani İngilizceye ciddi hakim olmanız isteniyor. (Pandemi nedeni ile şu an step 2 cs uygulanmıyor) başarılar dilerim.Ahmet Şahin
Bu aşamaları geçince asistanlık için müracaat edebilme şansına sahip oluyorsunuz bu sınavları vermek size mutlak asistanlık hakkı tanımıyor. Puanınıza ve asistanlık yapmak istediğiniz yere göre davet alabilir kabul görürseniz de asistanlığa başlayabilirsiniz. Henüz lisedeyseniz bunları planlamak daha kolay olacaktır, tıp fakültesi içinde çok son yıllara bırakmayın. başarılar.
Merhaba Cerrah olarak başlayabilmen için öncelikle USMLE Sınavlarını vermen ve yüksek puan alman gerekiyor. sonrasında referans mektubu ile nerede cerrahlık yapmak istiyorsan oraya müracaatını yapıyorsun. Müracaat ettiğin yerin kriterlerine uygun isen Görüşmeye davet ediyorlar, anlaşırsanız da cerrahi eğitimi başlamış oluyor.
@@hamza-ps2uy Step 2 CK dan sonra yani Step 2 CS ve diğer tüm sınav ve görüşmeler USA da yapılıyor, elbette yüksek puan alırsanız birden fazla yere müracaat yapabilir, birden fazla yerden kabul alabilir ve seçenekler içinde size en uygun olan yerde başlayabilirsiniz.
Şu nda okuduğum fakülte akredite fakat ben mezun olmadan önce akreditesi bitecek yine de akredite bir yerden mezun olmuş gibi bir diploma almış sayılır mıyım?
@@albusdumbledore9578 kardeş ne anladın sen merak ettim. Adam diyor ki tek kural işler step 1 e girdiğinde okulun akredite mi? Yani demek istiyor ki step 1 e başvuru yaptığında okulun akredite ise sonradan akreditasyon kalksa da önemli değil. Ben de diyorum ki ben okul akredite değilken mezun oldum okul sonra akredite oldu burda sorun çıkar mı?
Merhaba ; öncelile yorumunuz için teşekkür ederiz. Şu an icin bu tip yurt dışı ayağı olan bir hizmet vermiyoruz , Daha çok sınava nasıl çalışması gerektigi ile ilgili profesyonel danışmanlık veriyoruz. Sağlıkla kalın...
@@alpix8217 Merhaba tekrar, Ayrıca USMLE ile ilgili kurs programımız yok ancak TUS’a hazırlanmak USMLE ye hazırlanmak için doğru bir yönelim olabilir zira videoda da bertildiği gibi TUS konu dağılımı itibarı ile % 95 USMLE den uyarlanmıştır.
hocam usmle icin tip falultesi onemli midir bana anadolu da tip fakulteleri geliyo eger fark ederse mezuna kalicam cevaplarsaniz cok sevinirim 5 agustosta bitiyo surem
Merhaba, USMLE için mezun olunan okulun akreditasyonu olması isteniyor bu nedenle tercih yapacağınız üniversitenin öğrenci işlerinden bu bilgiyi edinebilirsiniz. Başarılar dilerim.
@@ahmetsahin8588 hocamm iyi geceler birde referans mektubu dediginiz ne oluyor arastirdim tekirdag namik kemalin akreditasyon programi varmis ama referans mektubunu profesorler mi veriyo ona gore universite bakicamda musait oldugunuzda cevaplarsaniz cok memnun olurum
@@kerimgun7582 Müracaat ederken sizden ABD de çalışan bir hekimden ( bu hekim Türk de olabilir ) yazılı sizi tanıdığını, çalışkan ve fayda sağlayacak biri olduğunuzu yazan belge. Yani sizi ABD de çalışan başka bir hekimden onay alma gibi düşünebilirsin.
Merhaba, bildiğim kadarı ile akredite değil yine de en sağlıklı sonuç için imedidata.com sitesinden kontrol edebilir veya dekanlıktan bilgi alabilirsin. Başarıla dilerim. Ahme Şahin
Merhaba, Genel cerrahi branşıda sınav sonrasında seçilmenize etken olabilir ancak doğrudan genel cerrah olamasınız. Yini bulundugunuz kurmunun kendi içindeki asistanlık ,uzmanlık sinsilesine uymanız gerekiyor.
TUSEM ANKARA peki abd’de çalışan bir türk hekimini nereden bulacağız? Öğrenci değişim programları gibi şeylerle mi? Bu kadar kısa süre tanıdıkları bir öğrenciye referans mektubu yazarlar mı?
@@zeyneplala1806 illa Türk olması gerekmiyor, staj yaptıgınız hastanedende alabilirsiniz, alamıyorsanız zaten Amerikada doktor olmanız konusunda eksikler gormus olabilirler, sonucta referans mektubu ne demek gördük begendik referasn oluyoruz demek buda size bağlı bir durum. başarılar dileriz.
Merhaba, Amerika’da asistan doktor ücretleri yıllık olarak belirlenir ve eyaletlere göre bölümlere göre farklılık gösterebilir. sağlık sigortası, ücretsiz yemek hakkı bazı eyaletlerde kira yardımı, kongre yardımı, kitap yardımı gibi yardımlar dışında vergiler düşündükten sonra 45.000 ila 60.000 $ arasında değişiyor. Başarılar dilerim.Ahmet Şahin.
Merhaba, diş hekimliği mezunları için okulunuzun ilk olarak ABD Eğitim Bilimleri departmanına sizinle ilgili transkript göndermesi gerekir bu belgeyi şahıs olarak gönderme şansınız yok, incelenen transkript ABD eğitim bilimleri kriterlerine göre yeniden puanlamaya tabi tutulur ve sizin kabul yada reddinize karar verilebilir. kabul edilen müracaatlardan ayrıca fark dersleri istenebilir. USMLE sınavları ise sadece tıp mezunları içindir. Başarılar dilerim.
Tekrar merhaba, Türkiye’de tıp eğitimi almış bir hekimin, ABD’ye uzmanlık için gelmeye karar vermeden önce , uzmanlık sonrası dönemi de mutlaka düşünmesi gerekmektedir. Uzmanlığını bitirdikten sonra dönmeyi düşünen ya da dönme yolunu açık bırakmak isteyen hekimlerin, uzmanlaşmak istedikleri alanın Türkiye’deki durumunu da dikkatle incelemeleri çok önemli bir basamağı oluşturur. Ülkemizdeki uzmanlık mevzuatı ve denklik kriterleri cumhuriyet tarihimiz boyunca birçok kez değişikliğe uğramıştır. Ufak farklılıklar genel olarak aşılabilse de, uzmanlık eğitimi süresindeki büyük farklılıkların aşılması mümkün olmayabilir. O nedenle ABD’de uzmanlık yapmak isteyen hekimlerin bu yola girmeden önce , seçtikleri uzmanlık alanının bu yönden karşılaştırmasını mutlaka detaylı bir biçimde yapmaları gerekir. Aksi halde ileride geri dönülmez yollara girmek pekala mümkündür ve işe başlarken henüz karar vermiş olmasanız bile dönme yolunu açık bırakmak ve ona göre planlar yapmak daha yerinde bir yaklaşım olacaktır.
Hocam diyelim ki bu sınavı kazandık kabul edildik aradan 2 3 yıl geçtikten sonra dönmeye karar verdik Türkiyede mesleğimize kolayca devam edebiliyor muyuz?
Yurtdışında tıpta uzmanlık eğitimi almış doktorların Türkiye’ye döndüklerinde de denklik almaları gerekmektedir. Bu durumda ise yurtdışında denklik için gidilen kurumun tanınmışlık listesinde olup olmaması, Uzmanlık eğitiminde geçirilen süre gibi farklı etkenler söz konusudur. Bu durumlara göre farklı denklik prosedürleri işlemektedir. Uzmanlığını bitirdikten sonra dönmeyi düşünen ya da dönme yolunu açık bırakmak isteyen hekimlerin, uzmanlaşmak istedikleri alanın Türkiye’deki durumunu da dikkatle incelemeleri çok önemli bir basamağı oluşturur. Ülkemizdeki uzmanlık mevzuatı ve denklik kriterleri cumhuriyet tarihimiz boyunca birçok kez değişikliğe uğramıştır. Ufak farklılıklar genel olarak aşılabilse de, uzmanlık eğitimi süresindeki büyük farklılıkların aşılması mümkün olmayabilir. O nedenle ABD’de uzmanlık yapmak isteyen hekimlerin bu yola girmeden önce , seçtikleri uzmanlık alanının bu yönden karşılaştırmasını mutlaka detaylı bir biçimde yapmaları gerekir. iyi Çalışmalar, Başarılar.Ahmet Şahin
Merhaba, bunun için başvuru yapacağınız klinikleri ve hocaları araştırmanız ve hangi konularda çalıştıklarını tespit etmek, o konuya duyulan alakayı doğru anlatıp birlikte çalışma istediğinizi belirten mailler atmanız gerek, hocaların bu mailler alışkın olduğunu ve sistemin bu mailler üzerinden döndüğünü hatırlatmak isterim. Başarılar dilerim.
@@ahmetsahin8588 Teşekkür ederim daha yolun col basindayim 5. Sinifim dili gelistirdikten sonra steplere baslayacam keske dili daha erkem gelistirseydim de simdiden match icin cv ye bir şeyler yapabilseydim
Merhaba, son bir yılda değişiklik olamadı ise akredite olmayan fakültelerin mezunları uluslararası sınavlara alınmayacak. Bu karar çok önceden alındı. Başarılar dilerim.
Hocam mezun olduğumuz üniversite akredite değilse denklik belgesi diye bir belge varmış. Fakülteden talep edebiliyormuşsun. Denklik belgesi akredite belgesi ile aynı işlevi mi görüyor tam olarak nedir nasıl alınır
Merhaba Denklik belgesi yurt dışında fakülte bitirenlerin mesleği Türkiye'de yapmak istediklerinde diplomalarını geçerli saydırmak için YÖK e başvurarak almaya çalıştıkları belgedir bu belgeyi almak için tıp fakültesi mezunları ÖSYM müracaat eder ve STS seviye tespit sınavına girerler, sınavda başarılı olanlara YÖK tarafından denklik belgesi düzenlenir. USMLE sınavı ise Türkiye'deki tıp mezunların Amerika'da denk sayılabilmesi için yapılan sınavdır, akreditasyon ise bu sınavlara girebilmeniz için ABD nin tıp fakültelerinin yeterliliği ile ilgili uyum kriterine verdiği isimdir. Üniversitelerin akreditasyon taleplerini ABD tarafından yetkilendirilmiş kişi ve kuruluşlar denetler ve onaylar. Denklik belgesi ile akreditasyon belgesi birbirinden ayrı kavramlardır ve aynı işlevi yaptığından söz etmek mümkün değil. Başarılar dilerim. Ahmet Şahin
Defne Hanım Cevaplandı.
Öğrenebildin mi nasıl yapıldığını
@@TUSEMANKARA_TUS hocam akredidasyon belgesi bizim fakültemizilde yok
Sıkıntı olurmu tşk
Hocam acaba Kanada içinde detaylı bir video yapabilir misiniz ?
Şu anki programlarımız daha çok yurt geneli kapsamlı ilerleyecek, teşekkür ederiz.
@@TUSEMANKARA_TUS ilginize ben teşekkür ederim
merhaba hocam,ilk once bilgisayarimin klavyesi turkce olmadigi icin kusura bakmayin lutfen. bi iki tane sorum var cevaplarsaniz sevinirim.
1.Bildigim kadariyla usmle toplam 4 sinavdan olusuyor, 2si ankarada ,3 u amerikada, birinci sinavi gecince 7 seneye kadar gecerli dediniz, peki 3. veya 4.sinavi gecemezse seneye hangi sinavdan girebilecek yani tekrar kaldigi sinavdan mi girmeye basliyacak YOKSA tekrar 2.sinavdan mi baslayacak Yani 2. 3. 4. sinavlarin da TIPKI 1.sinav gibi belli bir gecerlilik suresi var midir?
2.sorum:USMLE'nin tum 4 sinavini da gectikten sonra istegimiz bolume uzmanlik icin hangi asamalari gecmemiz gerekiyor?
TESEKKURLER
Merhaba,Bilgilendirmen için teşekkürler.
Sınavlarım ikisi Ankara’da diğer ikisi Amerika’da yapılıyor her birinin geçerlilik süresi yedi yıldır yedi yıl bitiminde daha önce girdiğiniz sınav bir sonraki Sınav için gerekli basamağı oluşturamaz yani geçerliliğini yitirir. Çalışmalarınızda başarılar dilerim Ahmet Şahin.
Merhaba hocam şu an sınavla ilgili tek değişiklik step 2 cs nin kaldırılması mı acaba? Step 3 var yine değil mi
Merhaba, doğrudur, mevcut durumda step 3 devam ediyor, step 2 CS ise belirsiz bir süre için kaldırıldı, bunun yanı sıra, gerek puan hesaplama gerek sınav formatı gerekse sınava hazırlanmak amaçlı dijital kütüphane (online) gibi çalışmalar olduğu ifade ediliyor ancak bu değişikliklerin hangi zaman diliminde hayata geçirileceği ile ilgili henüz netleşmiş bir tarih söylenmiş değil. Başarılar dilerim.
Türkiyede uzman olduktan sonra usmle ye girebiliyor muyuz hocam ? Bir de amerikada doktorluğa başladığımız zaman orda belirli bir süre kalma zorunluluğumuz var mı, mesela 10 yıl sonra tekrar Türkiyeye dönebilir miyiz
Akredite olmuş bir fakülteden mezun iseniz Türkiye de uzman olduktan sonra da USMLE ye katılmanda bir engel yok.
ABD de sınavları kazanıp uzman olarak çalışmak isterseniz bilmeniz gereken kontratları yıllık yapıldığı yani isterseniz kontrat bitiminde dilediğiniz yere gitme şansınız var. Başarılar dilerim.
@@ahmetsahin8588 çok teşekkür ederim :)
Yorumunuz için Teşekkürler.
Hocam öncelikle biz bu sınavları geçip asistan olarak göreve başladıktan sonra uzmanlığımızı amerikada mı alacağız ve asistanlıktan sonra amerikada çalışabilecek miyiz yoksa çalışma iznimiz iptal mi olacak
Evet doğru, asistanlık bitince uzmanlık belgenizi ABD den almış olacaksınız. Uzman olduktan sonra size sağlık profesyoneli olarak çalışabilmeniz için yıllık kontratlar teklif edecekler bu zaman diliminde vize türünüz J1 dan H1'a değişecektir. ABD’de sağlık profesyoneli olarak çalışabilmek için öncelikle ABD’de bulunan bir hastane ya da kliniğin işveren sıfatıyla sizi işe alması gerekmektedir. Bunun ardından, söz konusu işveren H1B vizesi almanız için sizin adınıza başvuru dilekçesi sunabilir. Çalışmalarınızda başarılar dilerim, Ahmet Şahin.
Şuanda ben 11.sınıfım seneye sınava giricem sizce hangi üniversite bize daha çok fayda sağlar akredite olması lazım dediniz bir de. Mesela Marmara üniversitesinin akreditasyonu bitecekmiş birkaç yıla bittikten sonra ne olur ? Çapa mı Cerrahpaşa mı yoksa Marmara mı bizim önümüzü daha çok açar? Sınavlara girdik diyelim yani USMLE ye kazansak hem step 1 i hem de step 2 yi sonra nasıl bir yol izlicez direk çalışabilecek miyiz? Bu süreçte neler yapmak bize fayda sağlar neler daha çok öne geçmemizi sağlar Amerika 'da doktorluk için?
Merhaba Yasemin, Üniversite anlamında sıralaman: Cerrahpaşa, Çapa ve Marmara şeklinde olabilir. Akreditasyonu biten üniversite büyük olasılıkla yeniliyor. Zor olan akredite olmak kolay olan süreyi uzatmak bu durum üniversitelere de prestij sağlıyor ve dolayısıyla hiçbir üniversite akreditasyonu iptal etmez. Cerrahpaşa dememdeki gerekçe ise: çalışma anlamında öğrencilerinin mümkün olduğu kadar önünü açıyor. Sınava nasıl çalışacağına gelince dönem birden itibaren temel bilimler ve temel bilimlerin TUS formatına dönem üçten sonra klinik bilimler ve günlük bilimlerin TUS formatını birlikte götürmen USMLE için kolaylık sağlayacaktır. Başarılar dilerim, Ahmet Şahin.
@@TUSEMANKARA_TUS Yanıtınız ve ilginiz için öncelikle teşekkür ederim. Bu dediğiniz 3 üniversiteden herhangi birini kazanıp USMLE ye girip onu da kazandıktan sonra bizi nasıl bir yol bekleyecek peki ? Direk çalışabilecek miyiz yoksa Amerika vatandaşı olmak gerekir mi uzun sure çalışabilmek için?
@@TUSEMANKARA_TUS Bir de aklıma takıldı biraz araştırma yapmıştım da Marmara nın tamamen ingilizce eğitim vermesi diğerlerine göre daha yeni olması gibi olanaklarının olması ile Cerrahpaşa dan ve Çapa dan daha iyi değil midir değişim programları da varmış baya emin olamadım ?
@@TUSEMANKARA_TUS merhaba, benim de bir sorum olacak. usmle'ye hazirlanmayi dusunen ve ingilizcesi cok iyi seviyede olan bir ogrenci icin turkce tip ve ingilizce tip tercihi arasinda bir fark var midir? tesekkur ederim.
@@maidebeydasinsi4053 umarım Türkçe seçmişsindir. İngilizce tıpın pek bir anlamı yok Türkiye’de
Merhaba tıp türkçe 2. sınıf öğrencisiyim. İngilizce seviyem iyi ama tıbbi bilgim çok düşük.Sizce step 1'e nasıl hazırlanmalıyım? Tus dershaneleri yardımcı olur mu ? Yoksa kendim piyasadaki kitaplardan mı çalışmalıyım?Ve step1 sınavına ne zaman girmeliyim.3. Sınıfın sonu iyi midir? 6 senenin sonunda uzmanlık kazanırsam türkiyede dondurup usmle hazırlıklarına devam edebilir miyim? Hem usmle'e hem de tus'a mezun olana kadar hazırlanabilir miyim?
Merhaba, Step 1 basic sciences; tıp eğitiminin ilk üç senesindeki temel bilimleri kapsıyor demek mümkün bu durumda sınava 3 ün sonunda yada 5 in sonunda step 1-2 ye beraber girebilirsin unutmaman gereken özellikle step 1 den yüksek skor yapman kabul için avantaj sağlar.
Kaynaklar için seçenek çok ister dersane kaynaklarından hazırlan ister USMLE World ve Kalan internet üyeliği ile sınavda benzeri olan sorulara ulaş veya internetten küçük bir araştırma ile kendine en uygun sitelerin kısa veya orta vadeli üyeliklerini tercih edebilirsin. Son soruna ise olumlu yanıt veremem Türkiye de uzmanlık dondurmak mecburi sebeplere dayandırılmalı raporlu sağlık sorunlar, doğum vb kısaca ikisine bir arada çalışılabilinir ancak ikisi bir arada yürütülemez. Ayrıca Türkiye deki uzmanlık ABD geçerli sayılmıyor. Tersi durum Türkiye için geçerli. Başarılar dilerim. Ahmet Şahin
@@TUSEMANKARA_TUS çok teşekkür ederim
Saygıdeğer hocam öncelikle yapmış olduğunuz videonuz için çok teşekkür ederim. Benim bir sorum var: Tıp fakültesine 21 yaşında başlamış olmam bir yıl hazırlık + altı yıl tıp sonucunda 28 yaşında mezun olmuş olacağım. 28 yaşımda mezun olacak olmam ABD'de doktor olmak için bana bir dezavantaj oluşturur mu?
rezervasyon
Merhaba, ben mezuniyet yaşınızın size dezavantaj olacağını düşünmüyorum 37 yaşında olup Amerika'ya giden olduğunu söylemem sanırım yeterli olur. Asıl olan sizin kendinizi nerede gördüğünüz, güveniniz kırılmasın. Başarılar dilerim. Ahmet Şahin
Hocam, step1’i mezun olduktan hemen sonra vermek istiyorum. Ama step 2 yi 2 sene sonra vermek istiyorum(Amerikaya yerleşmek, dili daha iyi öğrenmek ve para biriktirmek için). Bu mümkün mü? Bana eksileri olur mu?
Merhaba, evet mümkün ve dil yeterliliği konusu doğru düşünülmüş nihayetinde diğer steplerde çok ihtiyacın olacak, step 1 verdikten sonra yedi yol geçerlilik süren var dilediğin gibi kullanabilirsin. Başarılar dilerim. Ahmet Şahin
Hocam video harika olmuş, netleştirmek için soruyorum da, step 1 ve 2 bittikten 3 ü vermeden sonra direkt olarak Amerika'ya yerleşip asistan olarak başlayabiliyoruz değil mi? Ve hangi üniversitede asistanlığa başlayacağımız nasıl belirleniyor? Sınavları verdikten sonra tek tek okullara mı başvuruyoruz, kabul almama ihtimalimiz de var mı? ve herhangi bir asistanlık ücreti vize oturma izni vesaire çok uğraştırır mı?
Merhaba, öncelikle sürecin oldukça maliyetli ve zorlu olduğunu ve sonuca ulaşmanın çok kolay olmadığını tekrar belirtmek isterim, sorunuza gelince USMLE Step 3 Sınavı, USMLE serilerinin son sınavıdır. Bu sınavın diğerlerinden oldukça farklı özellikleri vardır. USMLE Step 3 sınavı, ABD’de tıpta uzmanlık eğitimine giriş için bir önkoşul değildir. USMLE Step 1, Step 2 CS ve Step 2 CK sınavları alan ABD’de tıpta uzmanlık yoluna girebilmeleri için yeterliyken, USMLE sınavlarının son basamağı olan Step 3 sınavı ABD’de kısıtlanmamış (unrestricted full medical license) eyalet lisansı alabilmek için gerekli bir sınavdır ve asistanlığa başlandıktan sonra alınabilir. Her eyalette tıbbi lisans alma koşulları farklı olduğundan, Step 3 ile ilgili değişik düzenlemeler vardır. Başarılar.
hocam, amerikada uzman olduktan sonra türkiyeye nasıl dönülür, tusa girmek gerekir mi? ya da önce türkiye de uzman olup amerikaya gitmek istesek usmleye girmemiz gerekir mi, girersek de türkiyede uzmanlığımızın olduğu için kabul daha kolay olur mu?
Merhaba ABD de yada Yurtdışında tıpta uzmanlık eğitimi almış doktorların Türkiye’ye döndüklerinde de denklik almaları gerekmektedir. Bu durumda ise yurtdışında denklik için gidilen kurumun tanınmışlık listesinde olup olmaması, Uzmanlık eğitiminde geçirilen süre gibi farklı etkenler söz konusudur. Bu durumlara göre farklı denklik prosedürleri işlemektedir. Uzmanlığını bitirdikten sonra dönmeyi düşünen ya da dönme yolunu açık bırakmak isteyen hekimlerin, uzmanlaşmak istedikleri alanın Türkiye’deki durumunu da dikkatle incelemeleri çok önemli bir basamağı oluşturur. Ülkemizdeki uzmanlık mevzuatı ve denklik kriterleri cumhuriyet tarihimiz boyunca birçok kez değişikliğe uğramıştır. Ufak farklılıklar genel olarak aşılabilse de, uzmanlık eğitimi süresindeki büyük farklılıkların aşılması mümkün olmayabilir. O nedenle ABD’de uzmanlık yapmak isteyen hekimlerin bu yola girmeden önce , seçtikleri uzmanlık alanının bu yönden karşılaştırmasını mutlaka detaylı bir biçimde yapmaları gerekir. Sorunun ikinci kısmını cevabı ise evet Türkiye'de uzmanlık alanlar ABD de yeniden uzmanlık almak durumunda, kısaca kabul etmiyorlar diyebiliriz.. Başarılar dilerim. Yeni videomuzda Amerika maaşları inceleyecegiz, takipte kalabilrsin, Ahmet Şahin
@@TUSEMANKARA_TUS çok sağolun hocam.
@@eminzorlu25 Rica ederim
Hocam merhaba akredite olmayan bir fakülteden akredite olan fakülteye geçiş yapıp oradan mezun olursak usmle'e girebiliriz değil mi
Merhaba, akredite olan bir fakülteden mezun olursanız USMLE ye katılmanız mümkün. Başarılar. Ahmet Şahin
Hocam anlamadığım bir konu var, usmle neticesinde yurtdisinda uzmanlık yaparken halihazirda doktor olarak çalışmıyor muyuz? Onların bize maaş vermesi gerekmiyor mu? Arkadaşların sorduğu eğitim ücreti nedir tam olarak
Merhaba, USMLE yi geçip asistanlığa başlarsanız aynen ifade ettiğiniz gibi onlar size ücret ödüyor, arkadaşlar sanırım Amerika'da tıp okumanın veya USMLE sınavlarına girmenin maliyetini sormuş olabilirler. Başarılar dilerim; Ahmet Şahin
Hocam diyelim ki bu sınavları geçtik hepsini ama Amerika'da referans olarak gösterebilecek bir tanıdığımız yok.
Bu durumda referans olarak bulabileceğimiz kişileri nereden bulabiliriz?
Bu kişilere internetten ulaşabilir veya yurt dışı öğrenci programları yapan şirketlerden yardım alabilirsiniz. Bu sınavları verene kadar mutlaka bir referans bulacağınıza inanıyorum. Ahmet Şahin.
Steplerin yıl içinde sınav tarihleri ve kaç kez yapıldığı ile alakalı bilgi bulamadım , bu konuda bilgilerinizi paylaşabilir misiniz ? Teşekkürler
Ömer bey, Amerikan kültür web sitesi yada arayarak bir bilgi kaynağı yada tarih sorabilirsiniz, usmle resmi sitesinde gerekli zamanlarda bilgi veriliyor.
Son aşamada kalırsak tekrar başa mı sarıyor sınavlar yoksa sadece ona mı giriyoruz tekrar
Merhaba, barajı geçen her sınavın geçerlilik süresi yedi yıldır. Süre sonunda sonuçlar sıfırlanır. Başarılar dilerim.Ahmet Şahin
Hocam merhaba, 3. Aşamaya gelenlerin yüzde kaçı bu sınavı geçiyor?
Merhaba, güzel bir soru ancak bu soruya yanıt verebileceğim bir istatistik yok elimde maalesef. Ancak tahminde bulun dersen dörtte bir diyebilirim. Başarılar dilerim. Ahmet Şahin
Çok teşekkür ederiz doğru düzgün bir USMLE tanıtım gördük sayenizde aklıma takılan 1-2 soru vardı yanıtlarsanız sevinirim Amerika’da girilen 2 sınav arası süre hakkında bilginiz var mıdır bir de referans mektubunu getirip sınavı geçtikten sonra ne zaman çalışmaya başlayabiliyoruz yanıtlarsanız çok memnun olurum saygılar
Merhaba ; Sınavlarınızı, belirli bir sistematiği olmakla beraber, kendi belirlediğiniz bir tarihte alabilirsiniz. Herhangi bir sınav sonucunuzun geçerliligieğeç geçtiyseniz yedi yıldır ve bu süre içinde aynı sınavı bir daha alamazsınız, geçemediyseniz yineleyebilirsiniz. ihtisas başvurusu her sene bir kez yapılır (Genelde Eylül 1 ile başlayan iki- üç aylık bir dönemde). Başlama ile ilgili tarih sizin başvurularınız ve bu başvurular için görüşme ve kabul aşamasından sonra gerçekleşir. Sorduğunuz bu zaman kişinin girişim becerisi ile uzayıp kısalabilir. Ancak görüşme öncesi dönem, görüşme günü ve görüşme sonrası bu dönemin asıl belirleyicisidir.Başarılar.
TUSEM ANKARA çok teşekkür ederim
Eğer bu sınavlara giriceksek Türkiye'de uzmanlık yapmak ne gibi bir fayda saglar
Furkan Emre Cuhadar memlekette kalır uzmanlık için TUS a girilir İngilizce temeli olmayanlar için risk bana kalırsa USMLE
@@furkanemrecuhadar4546 Türkiyede aldığınız uzmanlık USA da uzmanlıgı geçerli kılmaz ama kabül edilmenizde artı bir madde olarak işlev görür. Uzmanlıgı olmayan birine gore tercih sebebi olursunuz. Ayrıca Türkiyede Uzmanlık yapmak B planı olarak size bir güven verecek ve USMLE de beklediginiz sonucları alamazasınız burada bir garantiniz olacaktır, Başarılar.
merhaba tum adımları geçip gereken her şeyi tamamladığımız takdirde boşta kalma ihtimalimiz var mı acaba?en azından bir yerde işe başlamamız kesin mi yoksa hocam
Merhaba, tüm adımları geçmeniz gerek şart asıl olan müracaat yaptığınız yerlerden alacağınız olumlu yanıt işte bunun garantisi yok. Kısaca tüm sınavları vermek sizi mutlak iş sahibi yapar demek mümkün değil. Başarılar dilerim. Ahmet Şahin
Sinava turkiyede bir süre calisip girebiliyoruz değil mi yani üni biter bitmez girmeye gerek yok (sinav parasini karsilamak icin)
Merhaba
Evet sınava dilediğiniz zaman girebilirsiniz, Ahmet Şahin
Iyi gunler .sizi yeni izledim.burada uzman olan biri icin orda uzman olarak calismanin bir yolu varmidir? Ve ya universite mezunu ile uzman doktor arasinda bir fark varmidir asistanlik hayatinda ve ya sinavinda?
Merhaba, burada uzman olan biri için orda uzman olarak çalışmanın yolu videoda anlatılan sınav sisteminden geçmeniz ile mümkün. üniversite mezunu ile uzman doktor arasında fark sorusunun yanıtı, sınav sistem ve asistanlığa ulaşmak için izlenen yol anlamında hiçbir fark yok ancak asistanlık kabulünde öncelik sebebi olabilir, Türkiye'den uzmanlık alanların fazladan sahadaki tecrübe ve edinimleri asistanlıkta avantaj olacaktır. Başarılar. Ahmet Şahin
Merhaba hocam. Tusta fizyoloji, sinirbilim, mikrobiyoloji gibi temel bilim bölümleri seçilebiliyor. Anladığım kadarıyla USMLde bu yok. Türkiyede tıp okuduktan sonra ABD'de temel bilimlerden devam etmenin yolu nedir?
Merhaba, Öncelikle şu konuyu açıklayalım. Temel bilimlerde asistanlık diye bir kavram dünya da Türkiye'den başka bir ülkede yoktur. Türkiye temel bilimlere de TUS ile "asistan" almakta ve "doktora" programları haricinde bir yan yol yaratarak temel tıp bilimleri "uzmanlığı" gibi tuhaf bir kategori yaratmaktadır. Bu problem nedeniyle, dünyada fizyoloji uzmanlığı, biyokimya uzmanlığı gibi kavramlar anlaşılamamaktadır çünkü bu yollar tıpta akademik geleneklere aykırıdır ve anlamı yoktur.
Fizyoloji, biyokimya, anatomi, histoloji, farmakoloji gibi bölümlerin "asistanlığı (residency)" ve uzmanlığı olmaz.
Bu bölümlere girebilmek için "doktora (PhD) Doctor Of Philosophy" programları olur ve bitirene o konuda doktora verilir. Başarılar dilerim. Ahmet Şahin
@@TUSEMANKARA_TUS Anladım hocam, teşekkür ediyorum.
Hocam ve değerli arkadaşlar merhaba. Türkiyede uzman olduktan sonra, Amerikada aynı uzmanlığı yapmak için yine aynı sınavlardan geçiyoruz ve tekrar orada sıfırdan asistanlığa başlıyoruz değil mi? (yani burada uzmanlığı bitirmemiz bizim oradaki sürecimizi kısaltmıyor??) Yanlış biliyorsam bilenler aydınlatsın lütfen. Teşekkürler.
Merhaba Türkiye’de uzmanlık alan arkadaşlar Amerika’da uzmanlıklarını saydıramıyorlar. Asistanlık sürelerinden tedavi yöntemlerine kadar birçok fark neden gösteriliyor dolayısıyla her branşta baştan ihtisas istiyorlar. Başarılar dilerim, Ahmet Şahin
Malum ucretler biraz fazla. Sinava girmek için belli bir süre var mı? yani ben mezun olduktan sonra da girebilir miyim yoksa illa okurken mi girmem lazım?
Merhaba, çok haklısın ücretler konusunda, sınava belli bir sürede katılma zorunluluğunuz yok tabi ki ancak step 1 ve step 2 için mevcut okul bilgilerinin avantaj sağladığını bilmeniz yararlı olabilir, bunun dışında sınava mezun olduktan sonrada girebilirsiniz. Başarılar dilerim. Ahmet Şahin
@@TUSEMANKARA_TUS Hocam peki diğer ulkeler için de böyle icerikleriniz var mı? mesela Almanya'yi arastirdim. Bu şekilde bir sınavdan ziyade dil ön plana çıkıyor.(İngilizcem b1-b2 seviyesinde ama almancam hiç yok) diğer ülkelerle ilgili icerikler de gelebilir mi?nasıl gidilir, ne kadar para gerekli, bu parayı burs olarak alabilecegimiz bir yer var mı, tip için dili nereden ogrenebiliriz vs. tarzı. mesela kanadayi arastirinca gördüm ki doktorlar rahat ama oraya gidip doktor olarak calismak çok zormus.
@@TUSEMANKARA_TUS Çünkü artık ekonomik boyutunu geçtim ben mezun olana kadar ulkenin yarısı multeci olacak neredeyse.
Çok iyisinizz çok iyi videolarrr çok iyisinizz vücutlarınızz çokk iyii çok samimisinizzz çok iyi yayıncılarsınız ve çok iyi TH-camrlarsınız çok iyi oyuncularsınız MAŞALLAH çok yakışıklısınız ve çok güzelsiniz sizler buradaki insanlar gibi çok güzelsiniz ve çok yakışıklısınız vücutlarınızz çokk iyii teşekkürlerr ❤️❤️ çokk iyii vücüdunuzz varr çokk başarılısınızz çokk zekisinizz MAŞALLAH sizler buradaki insanlar gibi çokk başarılısınızz çokk zekisinizzz MAŞALLAH çok teşekkürler her durum için❤️❤️❤️❤️ İNŞALLAH ALLAH tüm insanların gönlüne göre versin AMİN
Teşekkür ederiz.
@@TUSEMANKARA_TUS Rica ederim ne demek
Değerli hocam eğer kısıtlanmamış eyalet lisansı almazsak yani step 3'e girmesek ABD'de doktorluk yapamaz mıyız ? Cevaplarsanız çok müteşekkir olurum şimdiden teşekkürler.
Merhaba, maalesef step lerin tamamını verdikten sonra diplomanız denk sayılıyor. Başarılar dilerim.Ahmet Şahin
@@TUSEMANKARA_TUS teşekkür ederim:)
Hocam nöroloji asistanlığı için Amerika'dan referans mektubu almak şart mıdır ? Yüksek USMLE puanı ile mektupsuz match alamaz mıyız ? Bir de referans mektubu şart ise kaçıncı sınıfta hangi şekilde (clerkship vb.) almalıyız ? Cevaplarsanız çok sevinirim hocam.
Merhaba, referans mektubu tüm asistanlıklar için gerekli sadece yüksek puan yeterli olmaz tüm bunlarla beraber müracaat yapacağınız yere detaylı bir öz geçmiş hazırlamanız sizden beklenenler arasında yayınlanmış en az birkaç makaleniz de varsa CV de belirtin tüm bunlar asistanlığa kabulünüz için garanti sağlamaz ancak kabul için avantaj sağlayacaktır. Başarılar dilerim. Ahmet Şahin
@@TUSEMANKARA_TUS Teşekkürler.
Hocam Step 1 in pass/fail only sistemine geçmesiyle birlikte alınan puanın önemi kalmadı mı yani sadece geçmek yeterli mi
Merhaba, bu konuda öngörüde bulunmak kolay değil ancak görüne o ki özellikle Türkiye gibi ülkelerden gidecek doktorların CV deki en önemli kozlardan bir tanesi gidecek gibi görünüyor, tabi bu kararın temelinde ABD li öğrencilerin bu yükten kurtulma planı olduğunu söylemek gerek. Türkiye'den gidecek öğrenciler için demek oluyor ki CV nin yayın ve ABD stajları bölümü daha değerli olacak. Başarılar dilerim.
Hocam uzman doktor maaşlarını 400bin dolar diye okudum acaba vergi kesintisiz hali midir yoksa bu meblağdan Yüzde 60falan vergi mi kesilicek?Ve sınav dışında yurt dışına gitme olasiligim düşük gibi duruyor referansım da yok acaba sıkıntı olabilir mi veya nasıl halledeilebilir ?cevaplayabilirseniz çok sevinirim
Merhaba, Genel olarak yurt dışındaki ücretler yıllık ve bürüt üzerinden konuşulur, yani elinize geçen net rakam değildir. Referans olmaması sorun olur tanıdık veya staj şansı ile bir şekilde referansı halletmen gerekir. Ahmet Şahin
%30 oranında vergi var. %60 değil. Ünsanlar Amerika'daki vergi oranını nedense çok abartıyor. Zanneden de maaşının %90'ı vergiye gidiyor sanıyor. Ama öyle birşey yok. Üstelik sene sonlarında vergi geri ödemesi alıyorsunuz. Bekarsan%30, evliysen%40 çocuğun varsa %50 oranında vergi geri ödemesi var.
Hocam pratisyen hekim olup amerikada pratisyenlik yapmak istiyorum. Olabiliyormu yoksa illa uzmanlıkmı istiyorlar.
Merhaba, Türkiye’de tıp okuyup USMLE sınavlarını geçerseniz ABD de eyaletlerde doktorluk yapabilme hakkı elde ediyorsunuz bu belge ile dilediğiniz eyalete müracaat edebilir ve kabul görürseniz aile hekim (pratisyen) olarak mesleğe başlayabilirsiniz. Başarılar dilerim.Ahmet Şahin
@@TUSEMANKARA_TUS Aile hekimliğini seviyorum hemde diğer uzmanlıklardan daha rahat çalışma konusunda. Yurt dışında aile hekimliği uzmanlığı yapıp ardından uzman olarak çalışabiliyormuyuz yani? İntarnette hiç bilgi görmedim aile hekimi olarak çalışan biri(yurt dışında)
Merhaba tavsiye mektubu olmadan pratisyen olarak calişabiliyor muyuz usmle steplerini geçince
on dokuz mayis universitesinin ingilizce programinin akreditasyonu var mi acaba? hicbir yerden bulamiyorum :(
Merhaba, on dokuz mayıs üniversitesinin akreditasyonu var olarak biliyorum ancak emin olmak için okul dekanlığından ve imedidata.com sitesinden kontrol edebilirsin. Başarılar dilerim. Ahmet Şahin
@@TUSEMANKARA_TUS turkce programinin varmis ama ingilizce programi yeni basvurmus akreditasyona (2019) . 6 yil icerisinde alabilir mi acaba akreditasyon?
Hocam yanlış mı anladım step1 ve step2yi geçtikten sonra orda doktorluk yapabiliyoruz,uzman olmak için step2öbür bölümü ve step3e girmek gerek
Merhaba, USMLE sınavı veya daha doğru bir deyimle “sınavları”, birinci basamak (Step 1), ikinci basamak teorik (Step 2 Clinical Knowledge), ikinci basamak klinik beceri (Step 2 Clinical Skills), üçüncü basamak (Step 3) olmak üzere 4 adet sınavın ortak adıdır. Bu sınavların her biri Amerika Birleşik Devletleri’nde herhangi bir eyaletin tıp doktoru lisansını alabilmek için (bağımsız pratik yapabilme hakkını elde edebilmek için) gerekli sınavlardır. Bu sınavların herhangi birini giren bir kişi hiçbir kadro alamaz, sadece gerekli basamaklardan birini tamamlamış sayılır. Diğer deyişle, USMLE sınavlarının tamamını tamamlamayan ne bir asistanlık programına başvurabilir, ne de eyalet lisansı alabilir. ABD’de asistanlık/uzmanlık eğitimine başvuran bir hekim, USMLE Step 1, Step 2 CK, Step 2 CS sınavlarını başarıyla geçmiş olmak zorundadır. ABD’de bir eyalette bağımsız çalışma izni almak isteyen bir hekim ise bu 3 sınava ek olarak USMLE Step 3 sınavını da başarıyla geçmiş olmak zorundadır. Dikkat edilirse, USMLE sınavlarının herhangi birisini ya da tamamını başarıyla geçmiş olmak, ABD’de hiçbir kadro garantisi vermez, hiçbir hastanede çalışmaya başlamak için bir pozisyonu sınav sonucuna bağlı olarak sunmaz. Ahmet Şahin.
@@TUSEMANKARA_TUS açıklama için teşekkür ederim hocam 🙏🏼
Hocam öncelikle oturum izni almalı mıyız yoksa bu sınavı kazanınca veriliyor mu ?
Merhaba, sınavı kazananların vizesi H1 dan J1 a çevriliyor, bu işlemleri kendiniz takip ediyorsunuz. Başarılar diliyorum.Ahmet Şahin.
Step 1 ve step 1 ck sınavlara sadece beşe kadar mı girebiliyoruz?
Hayır zaman sınırlaması yok.
Yurt dışı seçeneklerini değerlendirecek öğrenciler için altın değerinde bir video olmuş. Emeklerinize sağlık, teşekkürler. Ayrıca referans mektubu hakkında bir videonuz olursa çok mutlu olurum :)
Bora bey Merhaba, faydası olduysa ne mutlu. Referans mektubu USA'da staja gidildiginde o klinikte ki bir hekimden alınabiliyor. Aynı zamanda dünya çapında ünlü USA'da görev yapan bir cok Türk hekimi ünlüleri dahil olmak üzere mümkün olduğu oranda rereferans olma noktasında yardımcı olabiliyor. Biraz sosyal iletişim biraz doğru adresle çok zorlanılan bir konu olmuyor. Bu konu tek bir videoyu doldurmamız zor ancak sıkca sorulan sorular videosunda yer verilecek sorulardan biri olabilir, katkınız ve ilginiz için teşsekkürler. Nicelerinize referasn olmanız dilegiyle, hoşcakalın.
@@TUSEMANKARA_TUS Teşekkürler, iyi ki varsınız :D
@@suleymanborabozagac5378 Teşekkürler :)
Merhabalar, Amerika'da uzmanlık yaparken maaş alıyor muyuz ? Alıyorsak ne kadar bir ücret veriyorlar ? Teşekkürler..
Merhaba Amerika'daki asistan doktorlar yılda ortalama 60 bin dolar, uzmanlar ise ortalama 400 bin dolar maaş almaktadır.
Yillik 400bin dolar
@@abdurahmanhuseyin1693 efsane yaaaaa
@@azadcakmak6928 brüt maaş.
@@ornitorenkmafya2900 brüt olduğunu herkes biliyor. Vergisini çıkarınca yine de yüksek çıkıyor. Yorumunuzu okuyan da net maaşın %10 verginin%90 olduğunu zannedecek. Yorum yaparken düzgün yapın.
Amerika'da hangi uzmanlıklar olunur cerrahi bölümlerle ilgili birşeyler var diye duymuştum
Kazançları olarak soruyorsanız şu videomuza bakabilirsiniz. Ugur A. th-cam.com/video/x-tJftYGEhk/w-d-xo.html
@@TUSEMANKARA_TUS kazançlar değil mesele onlar sadece kendi milletlerine güveniyorlarmis ameliyat konusunda yani Amerikalı olmayan biri kalp beyin genel vb cerrah olamıyormus Amerika'da . Yani usmle sınavı bittikten sonra kişiler hangi uzmanlıkların asistanlığına girebiliyor
Yakın Doğu üniversitesinin akreditesi var öı acaba
Merhaba, bu bilgiye Yakın Doğu üniversitesi öğrenci işlerinden ulaşabilirsiniz. Başarılar dilerim
Hocam merhabalar. Hacettepe tıp 1.sinifim. Mezun olduktan sonra Amerika'da çalışmak istiyorum. Onun için her gün çalışıyorum ama bu sınavı kazanabilmek için günde kaç saat çalışmak gerekir
Merhaba, USMLE kazanmak için evet günde 1-3 saat ile başlayıp 5 sınıfın sonuna doğru 6-8 saate çıkabilirsen ciddi mesafe kat edersin ancak ne kadar çalıştığın sorusuna nasıl çalıştığın sorusunu da eklemelisin. Öncelik temel bilimler ve İngilizce iyileştirmesi olmalı 4. Sınıf ile klinik bilim eklemelisin. Başarılar dilerim. Ahmet Şahin
Hocam merhaba sabırla herkese cevap vermenizden cesaret alarak ben de sizden bilgi almak istiyorum ve çok teşekkür ederim. 6 aydır tıp fakültesinden mezunum pratisyen hekim olarak çalışıyorum 25 yaşındayım. ABD de doktorluk yapma şansımın olduğuna ikna oldum diyebilirim. Ve şartlarımı zorlamak istiyorum . Dil sevyem B2 . Sizce dili geliştirirken bir yandan da USMLE kaynaklarına mı hazırlanmalıyım ? Yoksa direkt USMLE kitaplarını tekrar ederek step 1 ve CK i geçebilir miyim ? En efektif çalışma nasıl olur bilmiyorum. Çevremde destek alacağım birileri yok. Bir yandan pratisyenlik yapıp bir yandan da çalışabilirim değil mi? TUS sınavı da iyi bir sonuç için çok efor istiyor. Uzun soluklu düşününce USMLE benim isteklerimi karşılıyor. Sadece yol yordam öğrenmek istiyorum. Dönerseniz çok memnun olurum. Saygılarımla
Kıymetli hocam bir de istediğiniz konu varsa paylaşın demişsiniz. USMLE step 1 ve 2 yi geçtikten sonra başvuruların kabulü için referans mektuplarına ihtiyacımız var bunun için de orda çalışmamız bu kısır döngü gibi geldi. Referans mektupları için yine kendimiz masraflarını karşılamak üzere orda bir üniversite gözlemci olarak çalışabilirmişiz . Ama bunun için nasıl kabul alacağım? CV de ne gibi şeyler sunmam gerek ki tabi gel gör desinler ?
Tekrar merhaba, bununla ilgili iki seçenekten ilki orada yaşayan ve çalışan Türk doktorlardan birilerine ulaşıp yardım alman, ikincisi ise öğrenci değişim programları yapan firmalardan ABD ye çalışan bir kurum bulman uygun olabilir.
İlave olarak bir üçüncü yol hangi branşta gözlemci olmak istiyorsan mail yolu ile müracatta bulunup kabul bekleyebilirsin.kolaylıklar başarılar.... Ahmet Şahin
Merhaba birkaç aşamada yanıt vermeye çalışacağım öncelikle hedef USMLE ise gerçekten çok iyi derecede İngilizce alt yapısına sahip olmak gerekir. Şöyle düşünme step 1 ve step 2 CK yı vereyim CS ve sonrası için dile yüklenirim bu yanlış olur çünkü step1 için yüksek puan gerekli olacak dolayısıyla iyi İngilizce orda da gerek.
Etkili çalışma için tek bir yöntemden bahsetmek doğru olmaz bu biraz kişisel ancak bana sorarsan, en uygun yöntem TUS çalışır gibi çalışman ve konu tekrarlarının ardından USMLE soruları çözmen bunu da her step için ayrı ayrı yapman ve tabi bol tekrar uygun olur. Başarılar Ahmet Şahin
Çok teşekkür ederim hocam saygılarımla.
@@kzlgezegen8708 teşekkürler, selamlar.
Hocam merhaba, sınavlar bittikten sonra başvurmak dışında asistan( yani maaş almaya başlamak) olmamız için yapmamız gereken başka bir prosedür var mı ?
Merhaba, USMLE Step 3 Sınavı, USMLE serilerinin son sınavıdır. Bu sınavın diğerlerinden oldukça farklı özellikleri vardır. USMLE Step 3 sınavı, ABD’de tıpta uzmanlık eğitimine giriş için bir önkoşul değildir. USMLE Step 1, Step 2 CS ve Step 2 CK sınavları alanABD’de tıpta uzmanlık yoluna girebilmeleri için yeterliyken, USMLE sınavlarının son basamağı olan Step 3 sınavı ABD’de kısıtlanmamış (unrestricted full medical license) eyalet lisansı alabilmek için gerekli bir sınavdır ve asistanlığa başlandıktan sonra alınabilir. Her eyalette tıbbi lisans alma koşulları farklı olduğundan, Step 3 ile ilgili değişik düzenlemeler vardır. Başarılar.
Hocam İngilizceyi az çok biliyorum ama tıpta normalde yabancı kelimeler var birde bunların İngilizcesini nasıl öğrencem
Merhaba, tıp fakültesine gelen arkadaşların büyük bölümü fen lisesi ve anadolu lisesi mezunları yabancı dil düzeyi çoğunda sizden farklı değil ve hepsi üniversitede ve devamında bu problemi çözüyorlar. Merak etmeyin kısa zamanda aynı kelimeleri duya duya alışıyorsunuz hatta bazen Türkçe karşılığını bulmakta zorlandığınız için günlük hayatta bu kelimelerden istemsizce yardım alıyorsunuz. Başarılar dilerim. Ahmet Şahin
Tşk.😀
ilk önce size çok teşekkür ederim, çok faydalı bir video oldu :) benim bir sorum var ; mezun olduktan sonra sadece step 1 ve step 2’yi yaptığımızda ve hem puanı yüksekse de referans mektubumuz varsa step 3 olmadan kabul edilebilir miyiz?ve uzmanlığa girip sonra step 3’ü bitirmemizlazım mı ? Birazcık kafa karıştırıyor 😄 . sınava 4.sınıfta hazırlamak ve 5.dönemin sonunda step 1 ‘e girmek geç mi hocam? Tekrardan teşekkürler 🙏🏻
Merhaba Dotchi Odeh ;
USMLE Step 3 Sınavı, USMLE serilerinin son sınavıdır. Bu sınavın diğerlerinden oldukça farklı özellikleri vardır. USMLE Step 3 sınavı, ABD’de tıpta uzmanlık eğitimine giriş için bir önkoşul değildir. USMLE Step 1, Step 2 CS ve Step 2 CK sınavları alanABD’de tıpta uzmanlık yoluna girebilmeleri için yeterliyken, USMLE sınavlarının son basamağı olan Step 3 sınavı ABD’de kısıtlanmamış (unrestricted full medical license) eyalet lisansı alabilmek için gerekli bir sınavdır ve asistanlığa başlandıktan sonra alınabilir. Her eyalette tıbbi lisans alma koşulları farklı olduğundan, Step 3 ile ilgili değişik düzenlemeler vardır. Dönem 5 sonu step 1 girmek konusu ; ne kadar erken o kadar iyi, ne kadar geç o kadar zor olur. Yinde kişisel tercihe kalmış bir durum. iyi Çalışmalar. Başarılar...
TUSEM ANKARA Yanıtladığınız için teşekkür ederim 🙏🏻 . ben şu anda 3.sınıftayım , 4.sınfta hazırlamayı başlayacam inşallah. tekrardan teşekkürler 😊
@@dianaodeh7005 Rica ederiz, Başarılar ...
Hocam yatay geçişle akredite olan bir üniversiteye geçilip ordan mezun olunursa usmle ye girilebilir mi
Merhaba Evet bunu yapabilirsen sınava girmende hiçbir engel yok.
TUSEM ANKARA teşekkür ederim
@@tzzt7182 Başarılar....
Hocam aile hekimi olmak için step 3'e girmemiz gerekiyor mu ve yine aile hekimliği için referans mektubu gerekli mi ? Cevaplarsanız çok sevinirim şimdiden teşekkürler.
Merhaba, evet aile hekimi olmak için de step lerin tamamını verdikten sonra step 3 de dahil diplomanız denk sayılıyor. Başarılar dilerim. Ahmet Şahin
@@TUSEMANKARA_TUS teşekkür ederim:)
hocam 11.sınıfı bitirip türkiyeden ingiltereye taŞındım ve ib sistem olarak 2 yıl okuyacağım sonrasında amerikada üniversite okumak istiyorum estetik cerrahiyi planlıyorum sat ye mi girmem gerekecek kaç yıl okuyacağım anlatabilir misiniz
Merhaba, Community College olarak adlandırılan iki yıllık okullara giriş şartları dört yıllık üniversitelere göre daha esnektir. Bu okullar SAT veya ACT sınav sonuçları istemezler. Ancak, değişmedi ise Dört yıl ve üzeri eğitim süreleri için üniversiteler için gerekli olarak biliyorum.
@@TUSEMANKARA_TUS sag olunnnnnn
Çok sağ olun tam olarak istediğim bilgileri vermişsiniz. İnşallah size yüzyüze de teşekkür etme fırsatı bulurum. Çok sağ olun
Merhaba, Faydalı olmasına sevindik. TUS ve Eksenindeki konular için Kurumumuza her zaman uğrayabilirsiniz , tanışmak icin randevulu gelirseniz daha garanti olur, iyi çalışmalar...
Hocam iyi günler Ben de yurtdışı düşünüyorum şuan üniversite tercih dönemindeyim.Şimdi benim şöyle bir sorunum var puanımın yettiği ve akredite olan ünilerin ingilizce hazırlığı yok ve benim İngilizcem 0 sizce tıpı Türkçe okusam kendimi bu sınavlara yeticek kadar ingilizce öğrenebilir miyim(İngilizcem berbat)
Merhaba, sıfır ingilizce ile başlayıp ileri düzey ingilizceye gelebilen bir çok örnek mevcut tabi ki sende yapabilirsin 1.ve 2. sınıfta yüklenmeye başlayıp USMLE sınavlarına kadar ileri düzeye gelebilmen mümkün. Başarılar dilerim.
@@ahmetsahin8588 teşekkür ederim.
Hocam ben 5. sınıfa geçiyorum, USMLE hazırlanmaya yeni karar verdim. TUS dershanesi başladığında dersleri dinlemem faydalı olacak mıdır yoksa sadece USMLE kaynaklarından mı çalışmamı önerirsiniz?
Merhaba. TUS dershanesinde dinlediğin dersler ve hatta biraz daha kolayı USMLE sınav soruları gibi düşünebilirsin, mutlaka faydasını göreceksin olur da fikrin değişir ve TUS a girmek istersen, eğitimin cebinde kalmış olur. Başarılar dilerim. Ahmet Şahin
Hocam merhaba.Türkiyede herhangi bir tıp fakültesinden mezun olunca bu sınavlara girmeye hak kazanabilir miyiz yoksa belli şartları var mı? Belirli üniversitelerden mi mezun olunmalı?
sartlardan biri akreditasyonu olan bi tip fakultesi
Merhaba, tek şart mezun olduğun üniversitenin akreditasyon sağlamış olması birçok fakültemiz bu durumu karşılıyor merak ettiğiniz fakülteleriimedidata.com adresinden sorgulayabilirsiniz. Başarılar dilerim. Ahmet Şahin
Teşekkürler cevaplarınız için
Hocam türkiye de uzmanlık alıp usmle ye girsek amerikada aynı dalda tekrar asistanlık yaptırırlar mı? Bu arada yararlı bir video olmuş teşekkürler
Tekrar merhaba, ABD de aynı dalda asistanlık yapabilmen için oluşturduğun dosyada CV kısmında Türkiye'de aynı bölümün asistanlığını yapmış olmanı belirtmen tercih sebebi olabilir, elini güçlendirir, daha kolay kabul görür. Hiçbir işe yaramaz demek mümkün değil ayrıca çokta zekice, ola ki bir gün Türkiye'ye dönmek istersen, hem ABD den hem de Türkiye'den uzmanlık almış olman, Türkiye'de tercih edilme konusunda lehine olacaktır,Ahmet.
TUSEM ANKARA teşekkür ederim
Hocam merhaba öncelikle video için çok teşekkürler
Hocam step1’e 5.sınıfın sonlarında katılmak için geç mi ? Neden?(şu an 4. Sınıfım)
Merhaba, Hayır geç değil dönem 4-5 ideal süreler ama daha fazla geciktirmemekte fayda var.
Merhabalar hocam, Alküde okuyorum 1. Sınıfım şuanda akredite degiliz akredite olmak için ne yapılmalı üniversitenin akredite olmaması durumunda akredite bir üniversiteye yatay geçiş yapmam bir sorun teşkil eder mi usmle için
Ben de Alkü de okuyorum ama geçiş yapmayı düşünüyorum eğer akredite bir üniversiteye geçiş yaparsan sınavla alakalı bir sıkıntı olmaz
Merhaba, akredite olmayan ünv. mezunları için sınava giriş mümkün değil koşulları yerine getirmesi gerek üniversite yönetimi yok onlar yerine getirmiyor ben yatay geçiş ile akredite olan bir ünv. ye geçebilirim diyorsan olur nihayetinde diplomanın alındığı okulun akreditasyonu sorgulanıyor. Başarılar dilerim.
@@naby569 teşekkürler
@@TUSEMANKARA_TUS teşekkürler
Merhaba, akredite olmayan ünv. mezunları için sınava giriş mümkün değil koşulları yerine getirmesi gerek üniversite yönetimi yok onlar yerine getirmiyor ben yatay geçiş ile akredite olan bir ünv. ye geçebilirim diyorsan olur nihayetinde diplomanın alındığı okulun akreditasyonu sorgulanıyor. Başarılar dilerim. Ahmet Şahin
Hocam ben şimdi 1.sınıf tıp öğrencisiyim diyelimki bu sınavları geçtim ABD'de asistanlığa başladım uzman olduktan sonra tekrar türkiyeye dönüp türkiyedr doktorluk yapmak istersem nasıl bir süreçten geçmem gerekiyor baştan uzmanlık yapmam gerekir mi? Eğer türkiyede yapılan step1 ve step2 sınavlarını geçıpte diğet sınavları geçemezsem sonra en başan tüm sınavlara girmek zorunda mıyım? Ayrıca bu sınavları kazanmak ne kadar zor? sonuçta çok pahalı sınavlar cevaplarsanız sevinirim şimdiden çok teşekkürler.
Merhaba ABD de yada Yurtdışında tıpta uzmanlık eğitimi almış doktorların Türkiye’ye döndüklerinde de denklik almaları gerekmektedir. Bu durumda ise yurtdışında denklik için gidilen kurumun tanınmışlık listesinde olup olmaması, Uzmanlık eğitiminde geçirilen süre gibi farklı etkenler söz konusudur. Bu durumlara göre farklı denklik prosedürleri işlemektedir. Uzmanlığını bitirdikten sonra dönmeyi düşünen ya da dönme yolunu açık bırakmak isteyen hekimlerin, uzmanlaşmak istedikleri alanın Türkiye’deki durumunu da dikkatle incelemeleri çok önemli bir basamağı oluşturur. Ülkemizdeki uzmanlık mevzuatı ve denklik kriterleri cumhuriyet tarihimiz boyunca birçok kez değişikliğe uğramıştır. Ufak farklılıklar genel olarak aşılabilse de, uzmanlık eğitimi süresindeki büyük farklılıkların aşılması mümkün olmayabilir. O nedenle ABD’de uzmanlık yapmak isteyen hekimlerin bu yola girmeden önce , seçtikleri uzmanlık alanının bu yönden karşılaştırmasını mutlaka detaylı bir biçimde yapmaları gerekir. Sorunun ikinci kısmını cevabı ise evet step 2 den sonra uzmanlık almak için diğer sınavlara girmen gerekir, barajı geçtiğin her sınavın geçerlilik süresi yedi yıldır. Diğer sorun ise evet sınavlar çalışmaya ve kişiye göre değişkenlik göstermekle birlikte kolay olduğu söylenemez ancak aynı zorluğun TUS içinde geçerli olduğunu söyleyebiliriz. Başarılar dilerim. Ahmet Şahin
@@ahmetsahin8588 çok teşekkürler hocam bundan daha güzel bir açıklama olamazdı
Bu bilgilendirici video için teşekkürler öncelikle. Akredite olmayan üniversite öğrencilerinin bu sınavlara girmesi için bir yol var mı acaba ?
Merhaba,
maalesef akredite olmayan üniversitelerin sınava girme şansı yok. Başarılar derim.Ahmet Şahin
hocam akreditesi olmayan bir üniversitede olursak veya girdikten sonra mezun olasıya kadar akreditesi biterse ve yenilemezlerse sınava giremiyoruz değil mi?
Merhaba, doğrudur ancak bildiğim akredite olup iptal ettiren fakülte şu ana kadar hiç olmadı. Başarılar dilerim. Ahmet Şahin
Video için teşekkürler.Ben Azerbaycanda 5 cı sınıf öğrencisiyim.Step 2 cs ankarada hazırlığı yapılıyormu?Yapılıyorsa ücreti ne kadar ?Diğer bir sorum referans mektubu için cerrahi branşda ne yapmamız gerekiyor?Yani mesela normalde hastaya dokunma iznimiz oluyormu o süreçde?
Önceden teşekkürler
Merhaba Sorduklarınızın bir kısmı video içinde yüzeyelde olsa değinildi, Ankarada bu tip bir hazırlık yok, aşamalara internet üzerinden geçiş yapıp referans mektubu aşamasında mezun olmus olup , ikili ilişkilerle amerikadaki bir doktorun size referasn mektubu vermesi gerekiyor.Başarılarınız, diyaloglarınız doğru kişilerle iletisim kurarak mektubu talep etmeniz süreci hızlandırabilir. Başarılar dileriz.
Hocam merhaba akredite olöayan bir fakültesinden mezun kişi üniversitenin daha sonra akreditasyon alması spnucu usml girme hakkı kazanır mı ?(örneğin mezun oldukdan 5 yıl sonra üniversite akretitasyon almış )
Akreditasyon aldı ise sorun olmaz, zaman önemli değil.
Bide seçilceğimiz kesin değil ortalama 35 bin lira gidiyo sizce pahalı değilmi siz ne düşünüyosunuz lütfen cevaplayın
Ücret i ödeyenlerin küçük bir yüzdesi bu sınavlarda başarı ve kabul yakalayabiliyor. Bir kesinliği yok sizin başarnıza bağlı. Ama kesiligi olsa evet hedefiniz yurtdışı ise değer bir ücret. Dİger yorum diş hekimligi demişsiniz, bu sınav sadece tıp hekimleri için diş hekimliği farklı onu araştırın istersiniz.
bütün sınavları verdiğin zaman giriyosun biyere nası seçilceğin kesin değil
merhaba ben ukrayna'da ingilizce tıp okuyorum.benim step 1'e nerde girmem gerekiyor?
Merhaba,
bildiğim kadarı ile Ukrayna’da step 1-2 ye girebileceğin merkez var yok ise Ankara’da girebilirsin. Başarılar dilerim.Ahmet Şahin
Hocam ben psikiyatrist olmak istiyorum Amerikada .bunun için anlattıklarınızdan başka bir şey yapmam lazım başka kaynak bulamadım cevaplarsanız sevinirim
Ne aşamadasın biraz kendinden bahsedersen faydası olabilir.
Hocam diyelim ki barajı geçemedik yeniden girmek için bir daha ücret ödememiz gerekir değil mi?
Merhaba, evet girdiğiniz tüm sınavların birbirinden bağımsız ayrı ayrı sınav giriş mevcuttur. başarılar dilerim.
Hocam diyelim tıp fakültesi 4. Sınıfta step 1'e girdik, mezun olana kadar step 2' ye girmeme gibi bir hakkımız var değil mi? 2 sene mezun olmak için bekleyebiliriz yani?
Merhaba Çok doğru step 1 sonrası, step 2 ye mezun olduktan sonra girebilirsin. Bilmen gereken sınavların geçerlilik süresi yedi yıldır, bu sürenin aşılmamasına dikkat etmelisin. Başarılar diliyorum, Ahmet Şahin.
@@TUSEMANKARA_TUS çok teşekkür ederim
@@emreyildirim6845 Rica ederiz
Hocam merhaba , Amerikada estetik ve plastik cerrah maaşı ne kadar ..
Merhaba ABD de kaç yıllık uzman olduğunuza ve performansınıza göre değişiklik göstermekle birlikte 150 bin ile 750 bin dolar arasında yıllık kontratlar konuşulması mümkün.
Videoda 235 referans dediniz referansları yayınlıyor musunuz ?
Merhaba
Referansları tek tek, yüz yüze eğitimde belirtiyoruz. Şu an için Ayrıca yazılı olarak yayınlamıyoruz.
Akredite olmayan bir fakülteden mezun olmak USMLE'ye girebilmek için bir engel teşkil eder mi?
merhaba, evet. Ancak akreditasyon zaman içinde alınabilir veya kaybedilebilir bir olgu. Yani şu an akredite olmayan br fakülte 3 sene sonra yeterlilikleri oluşturursa akredite olabilir
@@TUSEMANKARA_TUS yanıtınız için teşekkür ederim. Varsayalım ki akredite olmayan bir üniversiteden mezun olan bir öğrenci TUS'a girdi ve akredite bir üniversitede uzmanlık eğitimini tamamladı. Sonrasında herhangi bir nedenle Amerika'da çalışmak istediğinde bu mümkün olur mu? Olursa hekimi ne gibi engeller bekler?
@@cy3472 Daha detaylı bilgi için usmle resmi kurumları ile iletişime geçebilirisniz, üniversiteniz akredite degilse usmle ye kabulünüz olmayacaktır
@@TUSEMANKARA_TUS akreditasyon 2024 e kdr önemli değil o zamana kdr başvurursanız
@@alpix8217 Merhaba Okuyan arkadaşarında katkı alması adına açabilir misiniz, 2024 e kadar akreditasyon onemli degil derken ne demek istediniz?, Ugur.
Reference mektubunu Turkiyede ishlediyim hospital Dan ala bilirmiyim??
Merhaba, referans mektubunu ABD 'de çalışan bir hekimden alman gerekiyor. Başarılar.
Merhaba hocam, Türkiye'de ilk 6 senelik eğitimi akredite olmayan bir üniversiteden almış olup uzmanlığı ise akredite olan bir üniversitede okumuşsam yine de USMLE'ye girme hakkım olur mu?
Merhaba, asıl olan uzmanlığı aldığınız yer değil mezun olduğunuz tıp fakültesinin akredite olması. Başarılar.
Tamamdır hocam teşekkürler
Hocam sınavı kazanıp asistanlığı Amerika'da yapıp uzmanlığımızı orada alır ve çalışmaya başlarsak birkaç yıl sonra Türkiye'ye dönüp aynı uzmanlık burada geçerli olacak şekilde doktorluk yapabilir miyiz?
Merhaba ABD de yada Yurtdışında tıpta uzmanlık eğitimi almış doktorların Türkiye’ye döndüklerinde de denklik almaları gerekmektedir. Bu durumda ise yurtdışında denklik için gidilen kurumun tanınmışlık listesinde olup olmaması, Uzmanlık eğitiminde geçirilen süre gibi farklı etkenler söz konusudur. Bu durumlara göre farklı denklik prosedürleri işlemektedir. Uzmanlığını bitirdikten sonra dönmeyi düşünen ya da dönme yolunu açık bırakmak isteyen hekimlerin, uzmanlaşmak istedikleri alanın Türkiye’deki durumunu da dikkatle incelemeleri çok önemli bir basamağı oluşturur. Ülkemizdeki uzmanlık mevzuatı ve denklik kriterleri cumhuriyet tarihimiz boyunca birçok kez değişikliğe uğramıştır. Ufak farklılıklar genel olarak aşılabilse de, uzmanlık eğitimi süresindeki büyük farklılıkların aşılması mümkün olmayabilir. O nedenle ABD’de uzmanlık yapmak isteyen hekimlerin bu yola girmeden önce , seçtikleri uzmanlık alanının bu yönden karşılaştırmasını mutlaka detaylı bir biçimde yapmaları gerekir. Sorunun ikinci kısmını cevabı ise evet Türkiye'de uzmanlık alanlar ABD de yeniden uzmanlık almak durumunda, kısaca kabul etmiyorlar diyebiliriz.. Başarılar dilerim.Ahmet Şahin
evet denk
Değerli hocam. Dönem 1 öğrencisi USMLE sınavına nasıl hazırlanabilir? Bir de eğer yurtdışında doktor olunursa mezuniyetten sonraki zorunlu hizmet görevi yapılma zorunluluğu devam ediyor mu? Şimdiden çok teşekkür ediyorum.
Merhaba, Dönem bir öğrencisi normal eğitimle devam eder iken aynı zamanda temel bilimlerden soru çözmeli dönem üçten sonra klinik bilimler eğitimi ile birlikte Klinik soruları çözmeli yani özetle tıpta uzmanlık sınavına çalışır gibi çalışabilirsiniz nihayetinde sınav tipleri soru anlamında birbirine çok yakın.
Devlet hizmeti yükümlülüğüne gelince yurtdışında çalışacak isen yükümlülüğünü yerine getirmeden de gidebilirsin bağlayıcı bir durum yok. Sadece Türkiye’de olduğun süre içinde mesleği yapabilmen adına bu yükümlülüğü yerine getirmen bekleniyor. Başarılar dilerim. Ahmet Şahin
@@TUSEMANKARA_TUS Değerli Ahmet hocam çok teşekkür ediyorum.
Hocam burdan tıp okuduk usml sınavıda geçtik direk işe mi başlıyoruz amerika da yoksa bir okuma süreçi var mı
Merhaba, USMLE sınavı bilindiği gibi senin mezun olduğun tıp fakültesini diplomanın ABD de denk sayılması için yapılan sınavlar serisi bu seri bitirildikten sonra ister aile hekimi olmak üzere müracaat edebilir istersen diğer uzmanlık alanları için başvuru yapabilirsin belirtmeliyim ki bu sınavları geçmek veya başvuruları yapmak seni mutlak suret ile iş sahibi yapmaz. Asistanlığa kabul görmen durumunda yaklaşık dört yıl eğitim sürecinden geçersin fakat bu sürede aylık asistan ücreti almaya devam edersin. Başarılar dilerim. Ahmet şahin
@@TUSEMANKARA_TUS Hocam hangi üniversitlerin olduguna nerden bakarız Türkiyede
@@TUSEMANKARA_TUS Hocam sınavları geçtik diyelim Amerika da 4 sene daha mı okuyacağız
@@ashnotfound8219 Uzmanlık yapabilmen için en az dört yıl asistanlık eğitimi alman gerekiyor.
Hocam usmle sınavlarının ücreti 7k-10k dolar arasında mı birde geldiğimiz üniversiteye ne kadar ücret ödüyoruz yıllık yurdu içinde olursa teşekkürler
Merhaba, Bu okullara göre vesaire çok değişken bir konu o aşamalara gelindikten sonra kredi, burs başka seçeneklerde oluşabilir.
@@TUSEMANKARA_TUS çok teşekkürler
Hocam akredite olmayan tıp fakültesinden mezun olduysak usmleye giremez miyiz
Merhaba, akredite olmayan fakültelerin mezunları uluslararası sınavlara alınmayacak. Bu karar çok önceden alındı. Başarılar dilerim.
Diş hekimliği olabiliyomu hocam lütfen cevap verin
Cevap verdik diger yorumunuza.
hocam bir sorum var TR de tıp okuyup USMLE sınavlarındanda yüksek bir puan aldıktan sonraki süreç nasıl ilerliyor? ilginize teşekkürler.
Merhaba, step cs ve step3 e ABD de giriyor her ikisindede birebir hasta ile konuşma mülakatı yapılıyor bu bölümde daha çok İngilizce anlamanız ve konuşarak doğru yönlendirmeniz test esiliyor. Yani İngilizceye ciddi hakim olmanız isteniyor. (Pandemi nedeni ile şu an step 2 cs uygulanmıyor) başarılar dilerim.Ahmet Şahin
@@TUSEMANKARA_TUS hocam bu uygulamalarıda geçince artık USA da asistan doktor olarak çalışabiliyormuyuz
Bu aşamaları geçince asistanlık için müracaat edebilme şansına sahip oluyorsunuz bu sınavları vermek size mutlak asistanlık hakkı tanımıyor. Puanınıza ve asistanlık yapmak istediğiniz yere göre davet alabilir kabul görürseniz de asistanlığa başlayabilirsiniz. Henüz lisedeyseniz bunları planlamak daha kolay olacaktır, tıp fakültesi içinde çok son yıllara bırakmayın. başarılar.
Teşekkürler
abi bide ben üni den mezun olsam cerrah olarak usmle ye girebilyom mu girersem diyelim baya iyi not yaptım kendim mi başvuruyom onlar mı alıyo
Merhaba
Cerrah olarak başlayabilmen için öncelikle USMLE Sınavlarını vermen ve yüksek puan alman gerekiyor. sonrasında referans mektubu ile nerede cerrahlık yapmak istiyorsan oraya müracaatını yapıyorsun. Müracaat ettiğin yerin kriterlerine uygun isen Görüşmeye davet ediyorlar, anlaşırsanız da cerrahi eğitimi başlamış oluyor.
@@TUSEMANKARA_TUS görüşmeye oraya mi gidiyoruz bide en yüksek notu alsam yada full çeksem direk müracaat etsem 2,3 yere girme şansım nedir sizce
@@hamza-ps2uy Step 2 CK dan sonra yani Step 2 CS ve diğer tüm sınav ve görüşmeler USA da yapılıyor, elbette yüksek puan alırsanız birden fazla yere müracaat yapabilir, birden fazla yerden kabul alabilir ve seçenekler içinde size en uygun olan yerde başlayabilirsiniz.
Şu nda okuduğum fakülte akredite fakat ben mezun olmadan önce akreditesi bitecek yine de akredite bir yerden mezun olmuş gibi bir diploma almış sayılır mıyım?
Merhaba Bu konuda tek kural işler, ilk sınava yani step 1'e girdiğinde okulu akredite mi ? sonrasının bir önemi yok, başarılar.
@@TUSEMANKARA_TUS Ben okulu bitirdikten sonra okulum akredite olursa yine de sınava girebiliyor muyum? Yardımcı olursanız sevinirim.
@@hakancanl2367 kardeş adam yazmış ya işte. fakülten akredite olmadan step1e girersen sonrasının önemi yok
@@albusdumbledore9578 kardeş ne anladın sen merak ettim. Adam diyor ki tek kural işler step 1 e girdiğinde okulun akredite mi? Yani demek istiyor ki step 1 e başvuru yaptığında okulun akredite ise sonradan akreditasyon kalksa da önemli değil. Ben de diyorum ki ben okul akredite değilken mezun oldum okul sonra akredite oldu burda sorun çıkar mı?
siz mentorluk ve yardımcılık yapıyormusunuz usmle dershanenızde? yanı staj ayarlamaktır danışmanlıktır oraya gıdınce karşılanmaktır gıbı
Merhaba ; öncelile yorumunuz için teşekkür ederiz. Şu an icin bu tip yurt dışı ayağı olan bir hizmet vermiyoruz , Daha çok sınava nasıl çalışması gerektigi ile ilgili profesyonel danışmanlık veriyoruz. Sağlıkla kalın...
@@TUSEMANKARA_TUS sinava hazirliyor musunuz dershane gibi?
@@alpix8217 Merhaba tekrar, Ayrıca USMLE ile ilgili kurs programımız yok ancak TUS’a hazırlanmak USMLE ye hazırlanmak için doğru bir yönelim olabilir zira videoda da bertildiği gibi TUS konu dağılımı itibarı ile % 95 USMLE den uyarlanmıştır.
@@TUSEMANKARA_TUS uscamp sizin değil mi
@@alpix8217 Hayır, bizim degil.
hocam usmle icin tip falultesi onemli midir bana anadolu da tip fakulteleri geliyo eger fark ederse mezuna kalicam cevaplarsaniz cok sevinirim 5 agustosta bitiyo surem
Merhaba, USMLE için mezun olunan okulun akreditasyonu olması isteniyor bu nedenle tercih yapacağınız üniversitenin öğrenci işlerinden bu bilgiyi edinebilirsiniz. Başarılar dilerim.
@@ahmetsahin8588 tesekkurler hocam
@@ahmetsahin8588 hocamm iyi geceler birde referans mektubu dediginiz ne oluyor arastirdim tekirdag namik kemalin akreditasyon programi varmis ama referans mektubunu profesorler mi veriyo ona gore universite bakicamda musait oldugunuzda cevaplarsaniz cok memnun olurum
@@kerimgun7582 Müracaat ederken sizden ABD de çalışan bir hekimden ( bu hekim Türk de olabilir ) yazılı sizi tanıdığını, çalışkan ve fayda sağlayacak biri olduğunuzu yazan belge. Yani sizi ABD de çalışan başka bir hekimden onay alma gibi düşünebilirsin.
@@ahmetsahin8588anladim hocam cok tesekkurler
hocam izmir demokrasi tıp ın akreditesi var mıdır bi bilginiz var mı?
ay evet bilmek istiyorum
Merhaba, bildiğim kadarı ile akredite değil yine de en sağlıklı sonuç için imedidata.com sitesinden kontrol edebilir veya dekanlıktan bilgi alabilirsin. Başarıla dilerim. Ahme Şahin
teşekkürlerr
İngiltere de doktorluk ile ilgili bir video çekebilir misiniz
ileri ki zamanla için yapmaya çalışacağız..
Hocam merhaba eğer step 1 veya step 2’de başarısız olursak tekrar girme şansımız oluyor mu?
Merhaba , Başarısızlık durumunda 4 defa sınava girme hakkınız var. Umarız ilkinde herşey istediğiniz gibi olur.
TUSEM ANKARA teşekkürler
merhaba hocam genel cerrahi branşını kabül ediyorlarmı ?
Merhaba, Genel cerrahi branşıda sınav sonrasında seçilmenize etken olabilir ancak doğrudan genel cerrah olamasınız. Yini bulundugunuz kurmunun kendi içindeki asistanlık ,uzmanlık sinsilesine uymanız gerekiyor.
Referans mektubu nasıl alınabilir ki yurt dışında çalışmış hocalardan falan mı acaba?
Evet, ABD de çalışan bir Türk hekimden veya staj yaptıkları klinikten alabilirsiniz.
@@TUSEMANKARA_TUS teşekkürler bilgi için
@@ahmetc9305 Başarılar, sağlıklı günler
TUSEM ANKARA peki abd’de çalışan bir türk hekimini nereden bulacağız? Öğrenci değişim programları gibi şeylerle mi? Bu kadar kısa süre tanıdıkları bir öğrenciye referans mektubu yazarlar mı?
@@zeyneplala1806 illa Türk olması gerekmiyor, staj yaptıgınız hastanedende alabilirsiniz, alamıyorsanız zaten Amerikada doktor olmanız konusunda eksikler gormus olabilirler, sonucta referans mektubu ne demek gördük begendik referasn oluyoruz demek buda size bağlı bir durum. başarılar dileriz.
Gittiğimiz üniversitenin eğitim alırken akredite olması bu durum için bir engel mi yanıtlarsanız sevinirim
Merhaba, Okuduğunuz dönemde Akreditasyon alınması diplomanızın tanınması anlamını taşır. Başarılar...
Video icin tesekkurler
ingiltere icin de bilgi alabilir miyim. Orda uzmanlik yapmak istersek
İleri Zamanlarda videolar yapmayaya çalışacağız..
Hocam asistanken girebiliyor muyuz step 1e
Merhaba, evet girebilirsiniz, asistanlık yaptığınız dönem USMLE ye girebilmeniz için engel değil. Başarılar dilerim Ahmet Şahin
Hocam Ukrayna'da okuyorum tıp Amerika'da gidip çalışabilirmiyim çalışabilirsemdr nasıl
Merhaba, USMLE , Amerika dışında nerede okursanız okuyun ABD de doktorluk yapmak için olmazsa olmaz. Başarılar dilerim. Ahmet Şahin
Peki kolay iş bulabiliyor muyuz?
iş garantisi olan olan bir meslek dalı diyebiliriz.
asistan maaşları ne kadar abd'de?
Merhaba, Amerika’da asistan doktor ücretleri yıllık olarak belirlenir ve eyaletlere göre bölümlere göre farklılık gösterebilir. sağlık sigortası, ücretsiz yemek hakkı bazı eyaletlerde kira yardımı, kongre yardımı, kitap yardımı gibi yardımlar dışında vergiler düşündükten sonra 45.000 ila 60.000 $ arasında değişiyor. Başarılar dilerim.Ahmet Şahin.
Açık lise diploması sorun çıkarır mı
Merhaba hayır çıkarmaz, Tıp fakültesinden mezun olmanız, üstüne usmle yi geçmeniz başlangıç için yeterli olacaktır. Ahmet Şahin.
tıp türkçe okuyanlar için de geçerli mi
Merhaba, Evet Türkçe yada İngilizce okuyan aynı şartlar ile aynı sınavlara girerler. Ahmet Şahin
Diş hekimliği için de geçerli mi
Merhaba, diş hekimliği mezunları için okulunuzun ilk olarak ABD Eğitim Bilimleri departmanına sizinle ilgili transkript göndermesi gerekir bu belgeyi şahıs olarak gönderme şansınız yok, incelenen transkript ABD eğitim bilimleri kriterlerine göre yeniden puanlamaya tabi tutulur ve sizin kabul yada reddinize karar verilebilir. kabul edilen müracaatlardan ayrıca fark dersleri istenebilir. USMLE sınavları ise sadece tıp mezunları içindir. Başarılar dilerim.
tüm bu süreç ne kadar sürüyor
Merhaba, başlama sürene bağlı olarak değişkenlik olabilir. Tahmini süre ise normal koşullarda 1-2 yıl olabilir. Ahmet Şahin
abi amerikada doktor olup uzman olunca türkiyeye gelince o uzmanlık geçerli mi
Tekrar merhaba, Türkiye’de tıp eğitimi almış bir hekimin, ABD’ye uzmanlık için gelmeye karar vermeden önce , uzmanlık sonrası dönemi de mutlaka düşünmesi gerekmektedir. Uzmanlığını bitirdikten sonra dönmeyi düşünen ya da dönme yolunu açık bırakmak isteyen hekimlerin, uzmanlaşmak istedikleri alanın Türkiye’deki durumunu da dikkatle incelemeleri çok önemli bir basamağı oluşturur. Ülkemizdeki uzmanlık mevzuatı ve denklik kriterleri cumhuriyet tarihimiz boyunca birçok kez değişikliğe uğramıştır. Ufak farklılıklar genel olarak aşılabilse de, uzmanlık eğitimi süresindeki büyük farklılıkların aşılması mümkün olmayabilir. O nedenle ABD’de uzmanlık yapmak isteyen hekimlerin bu yola girmeden önce , seçtikleri uzmanlık alanının bu yönden karşılaştırmasını mutlaka detaylı bir biçimde yapmaları gerekir. Aksi halde ileride geri dönülmez yollara girmek pekala mümkündür ve işe başlarken henüz karar vermiş olmasanız bile dönme yolunu açık bırakmak ve ona göre planlar yapmak daha yerinde bir yaklaşım olacaktır.
Hocam diyelim ki bu sınavı kazandık kabul edildik aradan 2 3 yıl geçtikten sonra dönmeye karar verdik Türkiyede mesleğimize kolayca devam edebiliyor muyuz?
Yurtdışında tıpta uzmanlık eğitimi almış doktorların Türkiye’ye döndüklerinde de denklik almaları gerekmektedir. Bu durumda ise yurtdışında denklik için gidilen kurumun tanınmışlık listesinde olup olmaması, Uzmanlık eğitiminde geçirilen süre gibi farklı etkenler söz konusudur. Bu durumlara göre farklı denklik prosedürleri işlemektedir. Uzmanlığını bitirdikten sonra dönmeyi düşünen ya da dönme yolunu açık bırakmak isteyen hekimlerin, uzmanlaşmak istedikleri alanın Türkiye’deki durumunu da dikkatle incelemeleri çok önemli bir basamağı oluşturur. Ülkemizdeki uzmanlık mevzuatı ve denklik kriterleri cumhuriyet tarihimiz boyunca birçok kez değişikliğe uğramıştır. Ufak farklılıklar genel olarak aşılabilse de, uzmanlık eğitimi süresindeki büyük farklılıkların aşılması mümkün olmayabilir. O nedenle ABD’de uzmanlık yapmak isteyen hekimlerin bu yola girmeden önce , seçtikleri uzmanlık alanının bu yönden karşılaştırmasını mutlaka detaylı bir biçimde yapmaları gerekir. iyi Çalışmalar, Başarılar.Ahmet Şahin
Hocam mezun olup para ve dil kismini hallettim diyelim stepleri gectim match ici nasil staj bulabilirim.
Merhaba, bunun için başvuru yapacağınız klinikleri ve hocaları araştırmanız ve hangi konularda çalıştıklarını tespit etmek, o konuya duyulan alakayı doğru anlatıp birlikte çalışma istediğinizi belirten mailler atmanız gerek, hocaların bu mailler alışkın olduğunu ve sistemin bu mailler üzerinden döndüğünü hatırlatmak isterim. Başarılar dilerim.
@@ahmetsahin8588 Teşekkür ederim daha yolun col basindayim 5. Sinifim dili gelistirdikten sonra steplere baslayacam keske dili daha erkem gelistirseydim de simdiden match icin cv ye bir şeyler yapabilseydim
akredite olmayan üniversiterden amerikaya gitmenin başka bir yolu mevcut mu ?
Merhaba, son bir yılda değişiklik olamadı ise akredite olmayan fakültelerin mezunları uluslararası sınavlara alınmayacak. Bu karar çok önceden alındı. Başarılar dilerim.